大学英语三级历年考试真题汇总2009年12月英语三级考试A级真题及答案

大学英语三级历年考试真题汇总2009年12月英语三级考试A级真题及答案

Part II Structure (15 minutes)
  Directions: This part is to test your ability to construct grammatically correct sentences. It consists of 2 sections.
  Section A
  Directions: In this section, there are 10 incomplete sentences. You are required to complete each one by deciding on the most appropriate word or words from the 4 choices marked A), B), C) and D). Then you should mark the corresponding letter on the Answer Sheet with a single line through the center.
  16. We must find a way to cut prices ________ reducing our profits too much.
  A) without
  B) despite
  C) with
  D) for
  17. She didn’t know ________ to express her ideas in English clearly in public.
  A) which
  B) why
  C) what
  D) how
  18. ________ the weather improves, we will suffer a huge loss in the tourist industry.
  A) As
  B) Since
  C) While
  D) Unless
  19. We are happy at the good news ________ Mr. Black has been awarded the Best Manager.
  A) that
  B) which
  C) what
  D) whether
  20. It is important that we ________ the task ahead of time.
  A) will fish
  B) finished
  C) finish
  D) shall finish
  21. Would you please pass me the book ________ cover is black?
  A) which
  B) whose
  C) that
  D) its
  22. ________ in the company for three years, Mark has become experienced in business negotiations.
  A) Having worked
  B) Have been working
  C) Have worked
  D) Worked
  23. Not until she arrived at the meeting room ________ she had forgotten to bring the document.
  A) she realized
  B) did she realize
  C) she did realize
  D) does she realize
  24. John had never been abroad before, ________ he found the business trip very exciting.
  A) because
  B) though
  C) so
  D) while
  25. ________ some students are able to find employment after graduation, others will have to return to school and earn an advanced degree.
  A) Since
  B) While
  C) Because
  D) If
  Section B
  Directions: There are 10 incomplete statements here. You should fill in each blank with the proper form of the word given in brackets. Write the word or words in the corresponding space on the Answer Sheet.
  26. Employees are not allowed (make) ________ personal phone calls in the office.
  27. The shop assistant priced the goods before (put) ________ them on the shelf.
  28. The purpose of new technology is to make life (easy) ________, not to make it more difficult.
  29. The proposal about the annual sales (discuss) ________ at the next board meeting.
  30. Since we work in different sections of the company, we see each other only (occasional) ________.
  31. Some domestic manufacturers are busy increasing production, losing the chance to develop more (advance) ________ technology.
  32. I shall appreciate your effort in (correct) ________ this error in my bank account as soon as possible.
  33. If your neighbors are too noisy, then you have a good reason to make your (complain) ________.
  34. 30 percent of the students who (interview) ________ yesterday believe they should continue with their education until they have a university degree.
  35. Measures should be taken to avoid the negative effect (bring) ________ about by unfair competition.
  答案:16-20:ADDAC     21-25:BABCB
  26. to make 27. putting 28. easier
  29. will be discussed / is to be discussed / should be discussed
  30. occasionally 31. advanced 32. correcting
  33. complaint 34. were interviewed 35. brought

Part III Reading Comprehension (40 minutes)
  Directions: This part is to test your reading ability. There are 5 tasks for you to fulfill. You should read the reading materials carefully and do the tasks as you are instructed.
  Task 1
  Directions: After reading the following passage, you will find 5 questions or unfinished statements, numbered 36 to 40. For each question or statement there are 4 choices marked A), B), C) and D). You should make the correct choice and mark the corresponding letter on the Answer Sheet with a single line through the center.
  Google, the Internet search-engine company, has announced it will give more than twenty-five million dollars in money and investments to help the poor. The company says the effort involves using the power of information and technology to help people improve their lives.
  Aleem Walji works for Google.org — the part of the company that gives money to good causes. He said the company’s first project will help identify where
  infectious (传染性的) diseases are developing. In Southeast Asia and Africa, for example, Google.org will work with partners to strengthen early-warning systems and take action against growing health threats.
  Google.org’s second project will invest in ways to help small and medium-sized businesses grow. Walji says microfinance (小额信贷) is generally small, short-term loans that create few jobs. Instead, he says Google.org wants to develop ways to bring investors and business owners together to create jobs and improve economic growth.
  Google.org will also give money to help two climate-change programs announced earlier this year. One of these programs studies ways to make renewable (再生的) energy less costly than coal-based energy. The other is examining the efforts being made to increase the use of electric cars.
  The creators of Google have promised to give Google.org about one percent of company profits and one percent of its total stock value every year. Aleem Walji says this amount may increase in the future.
  36. The purpose of Google’s investments is to ________.
  A) help poor people
  B) develop new technology
  C) expand its own business
  D) increase the power of information
  37. According to Aleem Walji, the company’s first project is to ________.
  A) set up a new system to warn people of infectious diseases
  B) find out where infectious diseases develop
  C) identify the causes of infectious diseases
  D) cure patients of infectious diseases
  38. What kind of businesses will benefit from Google.org’s second project?
  A) large enterprises
  B) cross-national companies
  C) foreign-funded corporations
  D) small and medium-sized businesses
  39. From the fourth paragraph, we learn that Google’s money is also invested to help ________.
  A) start more research programs
  B) make more advanced electric cars
  C) develop renewable and coal-based energy
  D) conduct studies related to climate changes
  40. From the last paragraph we learn that the investments by Google.org come from ________.
  A) Google’s profits and stock value
  B) some international IT companies
  C) the company’s own interests
  D) local commercial banks
  Task 2
  Directions: This task is the same as Task 1. The 5 questions or unfinished statements are numbered 41 to 45.
  Your boss holds your future prospects in his hands. Some bosses are hard to get along with. Some have excellent qualifications but no idea when it comes to dealing with people. Of course, not all bosses are like that.
  The relationship you have with your boss can be a major factor in determining your rise up the career ladder. Your boss is not only your leader, he is also the person best equipped to help you do the job you are paid to do. He can inform you of company direction that may affect your professional development.
  Your boss also needs you to perform at your best in order to accomplish his objectives. He needs your feedback in order to provide realistic and useful reports to upper management. But how does this help you establish a meaningful working relationship with your boss?
  The key is communication. Learn and understand his goals and priorities (优先的事). Observe and understand your boss’s work style. If he has not been clear with his expectations, ask! Likewise, ask for feedback and accept criticism gracefully. And if he understands that you do not view your job as just something to fill the hours between 9 and 5, he may be more likely to help you.
  In short, getting along with your boss requires getting to know his likes and dislikes and learning to work with his personality and management style.
  41. The main idea of the first paragraph is that ________.
  A) bosses are hard to deal with
  B) bosses have good character
  C) bosses determine your career future
  D) bosses must have similar personality
  42. In the second paragraph, “rise up the career ladder” (Line 2) means ________.
  A) going to work abroad
  B) changing jobs frequently
  C) being promoted in position
  D) pursuing an advanced degree
  43. In order to achieve his objectives, your boss expects that you will ________.
  A) do your best in your work
  B) show your management skills
  C) get along with your colleagues
  D) write reports to upper management
  44. The most important factor for establishing a good working relationship with the boss is ________.
  A) high expectations
  B) quick feedback
  C) frequent criticism
  D) effective communication
  45. The best title for the passage might be ________.
  A) How to Take Care Of Your Boss.
  B) How to Get Along with Your Boss
  C) How to Accept Your Boss’s Criticism
  D) How to Accomplish Your Boss’s Objective
  答案:36-40:ABDDA   41-45:CCADB

Task 3
  Directions: The following is an announcement. After reading it, you are required to complete the outline below it (No.46 to No.50). You should write your answers briefly (in no more than three words) on the Answer Sheet correspondingly.
  We welcome you aboard the Eastern Flight and will do our best to make your trip comfortable and enjoyable.
  For your safety and convenience
  To begin the trip, we would like to draw your attention to some safety-related details. These are also explained on the instruction card in the seat pocket in front of you. Seat belts must remain fastened while the “Fasten seatbelts” sign is on. It is advisable to keep them fastened at all times while seated. All flights are non-smoking. The use of mobile telephones is now allowed when the airplane is on the ground. During the flight the use of CD and DVD players is not allowed.
  For your entertainment
  To help you enjoy your trip, we provide a range of newspapers. On our MD-11 and Boeing aircraft, we provide music and video programs. On Airbus A 321/320/319, short videos are shown.
  Meals and drinks
  During most flights we serve you a tasty meal and drinks. Beer, wine and other drinks are served free of charge. Coffee, tea and juice are served free of charge on all domestic (国内的) flights. On domestic flights leaving before nine and on all flights to Northern China, a snack is served.
  Eastern Flight Service
  Safety and convenience
  1) Seat belts: remain -46- while the “Fasten seatbelts” sign is on
  2) Smoking: not allowed on board
  3) Mobile phones: used only when the airplane is -47-
  4) CD and DVD: not allowed to play -48-
  Entertainment provided
  1) newspapers
  2) music and -49- on MD-11 and Boeing aircraft
  Meals and drinks on board
  1) meals served on most flights
  2) coffee, tea and juice served free of charge
  3) a snack served on all flights to -50-
  答案:46. fastened 47. on the ground 48. during the flight
  49. video programs 50. Northern China
  Task 4
  Directions: The following is a list of terms related to Security. After reading it, you are required to find the items equivalent to (与…等同) those given in Chinese in the table below. Then you should put the corresponding letters in the brackets on the Answer Sheet, numbered 51 through 55.
  A —————– air traffic control system
  B —————– armed police
  C —————– crime prevention
  D —————– entry requirement
  E —————– international criminal police organization
  F —————– level of security
  G —————– picket line
  H —————– police station
  I —————– patrolling vehicle
  J —————– safety precaution measure
  K —————– safety control device
  L —————– security command center
  M —————– security service
  N —————– security control center
  O —————– security personnel
  P —————– valid documents
  Q —————– security monitoring and control
  Examples: ( M ) 保安服务 ( G ) 警戒线
  51. ( ) 空中交通管制系统 ( ) 安全预防措施
  52. ( ) 巡逻车 ( ) 武装警察
  53. ( ) 国际刑警组织 ( ) 有效证件
  54. ( ) 入境要求 ( ) 安保人员
  55. ( ) 安全保障级别 ( ) 安全监控
  答案:51. A, J 52. I, B 53. E, P 54. D, O 55. F, Q
  Task 5
  Directions: The following is a business letter. After reading it, you should give brief answers to the 5 questions (No.56 to No.60) that follow. The answers (in no more than 3 words) should be written after the corresponding numbers on the Answer Sheet.
  Dear Mr. Smith,
  I am pleased to offer you the position of after-sales manager at our company starting on 16 June, 2009. I propose that the terms of employment will be those in the attached draft individual employment agreement.
  Please note that you are entitled to discuss this offer and to seek advice on the attached proposed agreement with your family, a union, a lawyer, or someone else you trust. If you want some information on your employment rights, you can also contact the Employment Service Office or visit our website.
  Also, if you disagree with, or do not understand or wish to clarify anything in this offer, please ring me to discuss any issue you wish to raise.
  If you are happy with the proposed terms and wish to accept this offer of employment, please sign the duplicate copy of this letter and return it to me by 1 June, 2009. In the event I have not heard from you by that date, this offer will be automatically withdrawn on that date.
  I look forward to working with you.
  Yours sincerely,
  John Brown
  56. What job position is offered to Mr. Smith in the letter?
  ___________________________________________________________.
  57. From whom may Mr. Smith seek advice about the proposed agreement?
  His family, a union, _____________________, or someone else he trusts.
  58. How can Mr. Smith get information about employment rights?
  By contacting the ___________________________ or visiting its website.
  59. When should Mr. Smith return the signed duplicate copy of this letter?
  By _______________________________________________________.
  60. What will happen if the duplicate copy of the letter is not returned by the deadline?
  This offer will be ___________________________________ on that date.
  答案:56. (The) after-sales manager 57. a lawyer 58. Employment Service Office
  59. 1 June, 2009 60. (automatically) withdrawn

Part IV Translation — English into Chinese (25 minutes)
  Directions: This part, numbered 61 through 65, is to test your ability to translate English into Chinese. After each of the sentences numbered 61 to 64, you will read four choices of suggested translation. You should choose the best translation and mark the corresponding letter on your Answer Sheet. And for the paragraph numbered 65, write your translation in the corresponding space on the Translation/ Composition Sheet.
  61. If either party wants to renew the contract, it should submit a written notice to the other party three months prior to the expiration of the contract.
  A) 如果任何一方希望撤销合约,必须将撤销的理由在三个月内通知对方。
  B) 如果合同一方希望重签合同,必须在合同到期三个月内写信通知对方。
  C) 如果任何一方希望更改合同,必须提前三个月向对方书面提交其理由。
  D) 如果合同一方希望续签合同,必须在合同期满前三个月书面通知对方。
  62. There is no sign that the world economic crisis will lessen in the next few months, although a certain degree of recovery is in sight.
  A) 尽管没有人认为未来几个月内世界经济危机会消失,但是在一定程度上的复苏是肯定的。
  B) 尽管世界经济复苏的迹象是肯定的,但是未来几个月内经济危机缓和的现象还不很明显。
  C) 尽管已经显现出一定程度的经济复苏,但没有迹象表明世界经济危机在未来几个月会减缓。
  D) 尽管没有人承认未来几个月内世界经济危机会触底,但我们肯定会看到世界经济的复苏。
  63. Most of the issues concerning personnel managementhave been solved satisfactorily; only a few of secondary importance remain to be discussed.
  A) 多数有关人员管理的问题顺利地解决了, 仅剩下几个问题还需要进行第二次讨论。
  B) 大多数有关人事管理问题已经得到圆满解决,只剩下几个次要的问题还有待于讨论。
  C) 很多有关人员配备问题基本上都得到了答复,只有第二个重要问题还未经过讨论。
  D) 第二个重要问题是有关人员调动的问题,这次已经得到妥善解决,不必再次讨论。
  64. Only in this way can Chinese enterprises improve their competitiveness and avoid being defeated by their foreign rivals after China’s entry into the WTO.
  A) 只有这个方法才能帮助中国企业去参加竞争,避免在加入世贸组织后被其外国对手所击败。
  B) 只有这样中国企业才能提高竞争力,并且在中国加入世贸组织后不会败给它们的外国对手。
  C) 如果中国企业要想在世贸组织中参加竞争,必须通过这种办法才能击败它们的外国对手。
  D) 中国企业只有通过这种途径来击败外国的对手,才能表明它们在世贸组织中具备竞争力。
  65. Thank you, Mr. Black. It’s a great honor to be appointed as Overseas Sales Manager. To be honest, this promotion came as quite a surprise to me. I’d like to think it’s mainly the contribution of the whole team. I’d like to thank all my colleagues in the company for their support and hard work. Due to their efforts, we’ve started some overseas projects successfully. Looking to the future, I’d still like to maintain contact with everyone, even though I’ll be working at the management level.
  答案:61-64:DCAB      65:谢谢您,布莱克先生。被任命为海外销售部经理,我感到极大的荣幸。坦白说,这次升迁令我十分意外。我想这主要是整个团队的功劳。我要感谢公司全体同仁们的支持与努力。由于他们的努力,我们海外的一些项目已经顺利的启动了。展望未来,尽管我将在管理层工作,我仍希望与大家保持联系。
  2009年6月英语三级考试A级真题及答案

Part II Structure (15 minutes)
  Directions: This part is to test your ability to construct grammatically correct sentences. It consists of 2 sections.
  Section A
  Directions: In this section, there are 10 incomplete sentences. You are required to complete each one by deciding on the most appropriate word or words from the 4 choices marked A), B), C) and D). Then you should mark the corresponding letter on the Answer Sheet with a single line through the center.
  16. By the end of this year Mr. Smith ________in our company for exactly three years.
  A) is working
  B) has worked
  C) will work
  D) will have worked
  17. I think that the Great Wall is worth ____________hundreds of miles to visit.
  A) to travel
  B) traveling
  C) traveled
  D) travel
  18. The new staff didn’t know how to use the system _________ I explained it to him yesterday.
  A) until
  B) because
  C) if
  D) since
  19. _____________is reported in the newspapers that the talks between the two companies have not made any progress.
  A) That
  B) What
  C) It
  D) As
  20. __________by the failure of the project,the manager could hardly say a word.
  A) To be shocked
  B) Shocked
  C) Be shocked
  D) Shocking
  21. The first question we now discuss is _______ early tomorrow.
  A) whether
  B) where
  C) what
  D) whom
  22. He was attending a meeting, ________he would have come to your party yesterday.
  A) unless
  B) when
  C) but
  D) or
  23. Enclosed you ________an application form that you are asked to fill out.
  A) will find
  B) find
  C) found
  D) are finding
  24. The auto industry spends large amounts of money on marketing campaigns _________young adult customers.
  A) attract
  B) attracted
  C) to attract
  D) attracts
  25. The advertising company recently hired a designer ____________had once won a prize in a national contest.
  A) whose
  B) which
  C) whom
  D) who
  Section B
  Directions: There are 10 incomplete statements here. You should fill in each blank with the proper form of the word given in brackets. Write the word or words in the corresponding space on the Answer Sheet.
  26. Your daughter is(luck)______enough to have large company.
  27. After an(introduce)______by the chairperson,we’11 go on with the day’s discussion.
  28. We must keep the manager(inform)___________of the advertising campaign.
  29. It is suggested that the president of the Union(make)_______a speech on behalf of all the workers.
  30. Having been badly damaged by the earthquake,the city has to be(rebuild) _______.
  31. She described the ancient city in detail because she(live)_________there for years.
  32. The new university graduate is confident of(win)________the post as the assistant to the managing director.
  33. Successful companies concentrate(much)_______on selling their products to their existing customers than to their new ones.
  34. The local economy depends(heavy)________on the goods.
  35. With such a short time(leave)______,it’s impossible for US to finish this complicated experiment.
  答案:16-20:DBACB         21-25:ADACD 
  26.lucky  27.introduction     28.informed    29.make    30.rebuilt     31.had lived
  32.winning     33.more     34.heavily     35.left

Part III Reading Comprehension (40 minutes)
  Directions: This part is to test your reading ability. There are 5 tasks for you to fulfill. You should read the reading materials carefully and do the tasks as you are instructed.
  Task 1
  Directions: After reading the following passage, you will find 5 questions or unfinished statements, numbered 36 to 40. For each question or statement there are 4 choices marked A), B), C) and D). You should make the correct choice and mark the corresponding letter on the Answer Sheet with a single line through the center.
  An ebook(also referred to as an electronic book,eBook,or e-book)is a digital version(版本)of a print book that you download and read. But if you want to read an ebook, you must have an Ebook Reader, which is a kind of free software used by your computer. Make sure you have installed the appropriate Reader before you download your ebook from the Internet. The software allows you to turn the words on the screen into the size you like. It also helps you turn pages and change your viewing options(计算机屏幕上的阅读选择). Ebooks are a fun alternative to regular books. You can download them to any computers and create your own library of hundreds of titles. If you load them onto your portable computer,you can take them with you when you travel. Some ebooks are even interactive! Best of all,when you order an ebook,there is no waiting and no shipping charges. The amount of time it takes to download your ebook depends on the speed of your connection and the size of your ebook.
  36. From this passage,we learn that an ebook___________.
  A) can be found in any library
  B) can be read directly from the Internet
  C) can be read by any one who has a computer
  D) can be read when special software is installed
  37. The Ebook Reader is used for___________.
  A) reading an ebook you’ve downloaded
  B) turning a print book into a digital version
  C) downloading an ebook from the Internet
  D) copying an ebook onto a portable computer
  38. From this passage,we can learn that______________.
  A) you can read an ebook on a laptop when you travel
  B) you Can order an ebook using the Ebook Reader
  C) the ebooks ordered have to be shipped to you
  D) it takes a lot of trouble reading an ebook
  39. Which of the following statements is TRUE?
  A) An ebook is ordered in print book is.
  B) The size of the words in an ebook cannot be changed.
  C) The downloading time is decided by the ebook’s size.
  D) There is less fun reading an ebook than a print book.
  40. The passage is mainly about_______________.
  A) a better way to download an Ebook
  B) a new kind of book—the Ebooks
  C) the new version of Ebooks
  D) the fun of reading Ebooks
  Task 2
  Directions: This task is the same as Task 1. The 5 questions or unfinished statements are numbered 41 to 45.
  Check-in Requirements
  Passengers and their baggage must be checked in at least 45 minutes before departure for domestic flights and 60 minutes for international flights. Government-issued photo identification is required for all passengers. Passengers traveling across any international boundary(边界)are responsible for obtaining all necessary travel documents. Passengers may be denied boarding if travel documents are not in order. When check-in requirements are not met,a passenger may be separated from his/her bag. Frontier Airlines will gladly hold the bag in the destination baggage service office for pick-up at the passenger’s convenience. Passengers must be at the gate 20 minutes before departure for boarding.
  Free Baggage Allowance for Each Ticketed Passenger
  Frontier Airlines,Inc. will accept,per ticketed passenger,two free checked bags not to exceed 62 inches and 50 pounds per piece—one carry—on bag and one personal item(purse,laptop,etc.).
  NOTE:A ticketed passenger may check one carry-on bag if it is within established limitations. All carry-on items must be put under a a shared overhead compartment(行李舱).
  Excessive(超重)Baggage Charges
  An excess charge will be made for each piece of baggage over the free allowance and for each piece of oversized or over-weight baggage.
  41. An international flight passenger should check in at least____________.
  A) 20 minutes before departure
  B) 30 minutes before departure
  C) 45 minutes before departure
  D) 60 minutes before departure
  42. What is required of all domestic and international passengers for check-in?
  A) An invitation letter.
  B) A company’s job offer。
  C) An official photo identification.
  D) An official immigration permit.
  43. How much baggage is a ticketed passenger allowed free of charge?
  A) One piece.
  B) Two pieces.
  C) Three pieces.
  D) Four pieces.
  44. What will passengers have to do if their baggage is over the free allowance?
  A) To pay extra money.
  B) To carry it themselves.
  C) To ask for special permit.
  D) To drop the excessive item.
  45. This passage is probably taken from_________.
  A) an advertisement of an airline
  B) a notice for airline passengers
  C) a schedule of an international flight
  D) an introduction to an airline company

答案:36-40:DAACB    41-45:DCBAB

Task 3
  Directions: After reading the following passage, you are required to complete the outline below it (No.46 to No.50). You should write your answers briefly (in no more than three words) on the Answer Sheet correspondingly.
  Sakura Scholarships offer students the opportunity of taking part in a three-month Japanese language course in the city of Kyoto. The scholarship covers free accommodation,meals and tuition. Return air fares between their country of residence and Osaka are paid,and $l,000 pocket money is also provided.
  To apply for one of these Scholarships,you should write a letter of about 300 words, describing your own educational career so far,and giving reasons education would benefit from participation in the Sakura Scholarship Scheme.
  The closing date for application is February l,2009.Applicants who have been selected for the short list(人选名单)will receive the notice by March 31. The final selection will be made on the basis of interviews held during May.
  Applications are open to all students,regardless of age, sex or nationality,and are also open to people who are not currently full-time students.
  Please apply to Ms. Kyoto Matsumoto,Sakura Scholarship Scheme,Sakura Trading Co, 200 East Avenue, London E97PS.
  Sakura Scholarships
  Course offered:Japanese language
  Items covered:
  1.accommodation,meals and -46-
  2. -47- air fares
  3. $l,000 -48-
  Content of application letter:
  1.describing the applicant’s -49- so far
  2.giving reasons for taking the course
  Deadline for application: -50-
  答案:46.tuition   47.return    48.pocket money   49.own educational career    50.February 1,2009  
  Task 4
  Directions: The following is a list of airport items. After reading it, you are required to find the items equivalent to (与…等同) those given in Chinese in the table below. Then you should put the corresponding letters in the brackets on the Answer Sheet, numbered 51 through 55.
  A—assembly line
  J—safety boots
  B—packer
  K—shipping clerk
  C—forklift
  L—time card
  D—explosive materials
  M—warehouse
  E—fire extinguisher
  N—hard hat
  F—loading dock
  O—earplug
  G—machine operator
  P—hand truck
  H—electrical hazard
  Q—safety earmuffs
  I—conveyor belt
  Examples: (F)装载码头 (K)运务员
  51.( )易爆材料 ( )安全耳罩
  52.( )]二时卡 ( )安全靴
  53.( )装配线 ( )灭火器
  54.( )安全帽 ( )手推车
  55.( )机器操作员 ( )输送带
  答案:51.D, 52.L,J 53.A,E 54.N,P 55.G,I
  Task 5
  Directions: The following is part of an introduction to life insurance. After reading it, you should give brief answers to the 5 questions (No.56 to No.60) that follow. The answers (in no more than 3 words) should be written after the corresponding numbers on the Answer Sheet.
  Dear Ms. Pascal,
  I would appreciate very much an opportunity to meet with you and demonstrate how my unique experience and education could be of particular advantage to your company’s future growth.
  My practice last summer at Pascal Business Systems helped me focus on my academic and career goals. My fourteen weeks with your company involved sales support activities including preparing brochures and catalogs,coordinating trade shows,providing data and information to salespeople in the field,and interacting with customers.
  During the past two semesters I have concentrated on developing my electronic publishing and business communication skills. I am confident that I Can improve the ways in which Pascal Business Systems provides support for field marketing representatives. This would include designing and maintaining an online catalog which could be coordinated with current inventories(库存清单).
  I am available for an interview at your convenience and look forward to an opportunity to discuss ways to improve operations and communications with field locations and sales. Thank you for your kind consideration.
  Sincerely,
  Lourdes Santiago
  56. How long did Lourdes Santiago work at Pascal Business System?
  ______________________________
  57. What activities was he involved in when he had his practice in the company?
  A whole variety of____________________________.
  58. What special skills did Lourdes Santiago develop during the past two semesters?
  He developed electronic publishing and_________________________
  59. What could he do to help support the field marketing representatives?
  By designing and maintaining__________________________
  60. What is ?
  Lourdes Santiago writes the letter to ask for__________________________.
  答案:56.Fourteen weeks    57.sales support activities    58.bussiness communication skills    59.an online catalog  60.an interview

 Part IV Translation — English into Chinese ( 25 minutes)
  Directions: This part, numbered 61 through 65, is to test your ability to translate English into Chinese. After each of the sentences numbered 61 to 64, you will read four choices of suggested translation. You should choose the best translation and mark the corresponding letter on your Answer Sheet. And for the paragraph numbered 65, write your translation in the corresponding space on the Translation/ Composition Sheet.
  61. For your safety,the electrical appliance can only been correctly mounted on a dry and cool base.
  A) 你只有在干燥通风的地方操作这一台电子仪器才安稳。
  B) 只有在干燥通风的场所才能使电子仪器持续运转。
  C) 为了安全起见,本电器设备只有安装在通风阴凉处才能确保其正常运行。
  D) 为了安全起见,本电器装置只有正确安装在干燥阴凉的基座上才能运行。
  62. Not only does asking questions at a meeting involve you in discussions,it is also a good way to demonstrate your professional knowledge.
  A) 会上提问不仅能使你参与到讨论中,而且还是一种展示你专业知识的好方法。
  B) 会上提问不仅可以讨论你所提出的问题,而且还可以很好地利用你的专业知识。
  C) 提问题不仅能使会议讨论你所关注的问题,还是一种表现你专业技能的好途径。
  D) 在会上不仅要提问还要参与讨论,这是提高你自己专业技能的一种有效方法。
  63. People feel that the interdependence of nations,SO long talked about by statesmen,is today more than ever a reality.
  A) 人们感到,一直都在谈论的国家的独立,今天比以往任何要。
  B) 人们感到,政治家长期谈论的国家间的相互依存,现在比任何时候都更为实在。
  C) 人们感到,许多国家以前一直关心的独立问题,现在比以前任何时候都更加现实。
  D) 人们感到,政治家以前认为国家之间要互相依赖,今天显得比以往更加现实了。
  64. The purchaser will not be responsible in connection with the packing or delivery of the above goods.
  A) 卖主并非没有责任解决上述货物在包装和运输方面出现的问题。
  B) 对于上述货物的包装和运输有关费用的任何事宜卖主概不负责。
  C) 以上货物在包装和运输方面所产生的有关费用买主均不予过问。
  D) 买主概不承担与上述货物的包装和运输有关的任何成本或费用。
  65. The city government recognizes that citizens have certain new needs. To better meet your needs, we have made several changes in community facilities in 2009.Three stations for the suburbs have been added to the western train service. Broadband(宽带)cable parts of the city. 5,000 new books were bought for the City Library. Some new facilities at the city hospitals have been installed. Next year, we will try our best to make your life even better.
  答案:61-64:DABD     65:市政府意识到居民有新的需求。为了更好地满足您的需求,我们于2009年已添加了新的社区设施。西去郊区的火车站已经增加了三站;现在城市的各个角落都设有宽带网;市图书馆购进5,000本新书;市医院安装了新的医疗设备。明年我们将尽力使您的生活更美好。
  

2008年12月英语三级考试A级真题及答案

Part II Vocabulary & Structure
  Directions: This part is to test your ability to construct grammatically correct sentences. It consists of 2 sections.
  Section A
  Directions: In this section, there are 10 incomplete sentences. You are required to complete each one by deciding on the most appropriate word or words from the 4 choices marked A), B), C) and D). Then you should mark the corresponding letter on the Answer Sheet with a single line through the center.
  16. Allan is looking forward to _______ the trade fair.
  A) meet
  B) meeting
  C) be meeting
  D) having met
  17. The sales manager had his secretary ________ a press conference for their new products.
  A) arrange
  B) to arrange
  C) have arranged
  D) arranged
  18. The message ________ Mr. Black was elected chairman of the committee arrived just in time.
  A) which
  B) what
  C) that
  D) how
  19. They had talked only for a few minutes ________ they found they were of different opinions.
  A) unless
  B) while
  C) before
  D) once
  20. By the time you month, I ________ my term paper.
  A) have completed
  B) complete
  C) am completing
  D) will have completed
  21. It is important to provide an environment ________ people are encouraged to make suggestions at all levels of the company.
  A) from which
  B) on which
  C) in which
  D) for which
  22. The scientists wanted to keep people ________ about the breakthrough in their experiment.
  A) inform
  B) informed
  C) informing
  D) to inform
  23. When I first arrived in Japan, I was surprised ________ the way people greeted each other.
  A) of
  B) to
  C) with
  D) at
  24. Not until that day ________ the importance of good manners in a job interview.
  A) did I realize
  B) I did realize
  C) I have realized
  D) have I realized
  25. The market economy is quickly changing people’s idea on ________ is accepted.
  A) that
  B) which
  C) what
  D) how
  Section B
  Directions: There are 10 incomplete statements here. You should fill in each blank with the proper form of the word given in brackets. Write the word or words in the corresponding space on the Answer Sheet.
  26. He tried to solve the problem, but he (quick) ________ gave up.
  27. The guest paid (little) ________ money than he should for the room.
  28. It is far more (interest) ________ for me to chat online with friends than to watch TV.
  29. Finally the woman found her (lose) ________ child with the help of the police.
  30. a raise in salary because of his excellent (perform) ________ at work.
  31. This hospital, which (equip) ________ with modern facilities, is one of the best in the country.
  32. Before (write) ________ an application letter, you should be aware what kind of people the employer needs.
  33. In the past few years, traffic problems (become) ________ more and more serious.
  34. It’s really (wonder) ________ to see you here again in Beijing.
  35. If I (be) ________ you, I wouldn’t miss the job interview tomorrow morning.
  答案:16-20:BACCD    21-25:CBDDC
  26.quickly 27.less 28.interesting 29.lost 30.performance
  31.equipped 32.writing 33.have become 34.wonderful 35.were

Part III Reading Comprehension
  Directions: This part is to test your reading ability. There are 5 tasks for you to fulfill. You should read the reading materials carefully and do the tasks as you are instructed.
  Task 1
  Directions: After reading the following passage, you will find 5 questions or unfinished statements, numbered 36 to 40. For each question or statement there are 4 choices marked A), B), C) and D). You should make the correct choice and mark the corresponding letter on the Answer Sheet with a single line through the center.
  The iMOVE database(数据库) information platform for persons interested in job opportunities offered by German companies. The information can be obtained in seven languages: German, English, French, Spanish, Chinese, Russian and Arabic.
  All programs in the area of professional training are carried out by German training providers. All these programs have a clear content to meet your special needs.
  Programs which take place in Germany are carried out in English. Many of the German training providers have started their training activities towards the international market. Therefore, they also offer courses abroad. These courses are taught in English or the language of the target country. Providers will be glad to supply you with additional information on these courses. You can contact the provider directly to find out more about a program and the training provider.
  To guarantee high standards in the database, iMOVE has developed quality standards for training providers and their services. All of the training facilities in the iMOVE database have to follow these quality criteria (标准). All training providers who publish their international training programs in the iMOVE database have recognized our General Terms and Conditions.
  36. The iMOVE database is intended for persons who ________.
  A) seek a job as a language translator
  B) are interested in the German language
  C) want to be employed by German companies
  D) wish to work for professional training providers
  37. The iMOVE programs are carried out by ________.
  A) language training centers
  B) German training providers
  C) special service developers
  D) overseas employment advisers
  38. held in Germany are taught in _________.
  A) German
  B) English
  C) French
  D) Chinese
  39. Which of the following measures has iMOVE taken to guarantee its high standards?
  A) Offering different language courses.
  B) Providing modern training facilities.
  C) Starting training courses overseas.
  D) Developing quality standards.
  40. The purpose of the passage is to ________.
  A) advertise the iMOVE database
  B) make German companies more popular
  C) hire overseas employees to work in Germany
  D) encourage people to learn more foreign languages
  Task 2
  Directions: This task is the same as Task 1. The 5 questions or unfinished statements are numbered 41 to 45.
  It is so easy to earn rewards!
  A wide range of partners wherever you go
  To make traveling even easier, Air France offers you Frequency Plus (常旅客里程奖励计划) through an extensive network of our partners. Together, they give you endless opportunities to earn Miles and a wide choice of Rewards, from airline tickets, hotel room-nights, car rentals and more.
  Partners in China
  If you are a Frequency Plus member registered in China, you have even more choices with rewards from our local partners. For more information, please contact your local Air France office.
  First rewards start as low as 20,000 Miles!
  Your Temporary Card
   that will make you get your first Reward, now.
  a) Show your Temporary Card and simply give your Card number when making a reservation and at check-in for your flight. In this way, you will begin to earn Miles even before receiving your Permanent Card.
  b) Fill in and return your personal membership (会员) application today. Your application must be received within 3 weeks of your first flight.
  Remember, you will enjoy an extra 3,000 Bonus Miles (额外奖励里程) for your first trip with Air France within 6 months of joining. Fly today and earn Rewards sooner!
  41. Air France carries out its Frequency Plus ________.
  A) through its reduced ticket prices
  B) through its extensive network of partners
  C) by providing convenient flight schedules
  D) by offering excellent services on board
  42. In order to get the first reward, passengers must fly at least ________.
  A) for 6 months
  B) 20,000 miles
  C) 3,000 bonus Miles
  D) once in three weeks
  43. Passengers can begin to earn Miles ________.
  A) if they reserve tickets at a hotel
  B) if they buy tickets at the airport
  C) before receiving the Permanent Card
  D) after receiving the Permanent Card
  44. Passengers should send their application to Air France ________.
  A) upon receiving the application form
  B) within 3 weeks of their first flight
  C) before their first flight with Air France
  D) within 6 months of joining the program
  45. The extra bonus Miles for a passenger’s first trip with Air France within 6 months of joining is ________.
  A) 2,000 miles
  B) 3,000 miles
  C) 20,000 miles
  D) 30,000 miles
  答案:36-40:CBBDA     41-45:BBCBB

Task 3
  Directions: The following is a Memo. After reading it, you are required to complete the outline below it (No.46 to No.50). You should write your answers briefly (in no more than three words) on the Answer Sheet correspondingly.
  The College of Business Management (CBM) is seeking new members to represent the college as Student Ambassadors (大使). will represent the college during the school year. Their term of service will begin during the autumn. Student Ambassadors will have the opportunity to cooperate with students at various activities throughout the school year.
  Purposes:
  1. Promote the College of Business Management.
  2. Improve leadership and communication skills.
  3. Assist in the recruitment of students to CBM.
  Qualifications and Requirements:
  1. Must currently be a full-time student in the College of Business Management.
  2. Must be available to assist with various recruitment activities.
  3. Must be able to contact students from various backgrounds.
  Application Procedure:
  1. Print a copy from our website.
  2. Complete the application form and return it to the Student Advising Office.
  For More Information:
  Contact Mark Davis at 472-2310 or stop by the Student Advising Office.
  Student Ambassadors Wanted
  Term of service: throughout the school year
  Tasks:
  1. represent the -46- as Student Ambassadors
  2. cooperate with students at -47-
  Purposes:
  1. promote CBM
  2. improve leadership and -48- skills
  3. assist in the recruitment activities
  Qualifications: must be a -49- student in CBM
  Application Procedure:
  1. print a copy
  2. complete the form
  3. return it to the -50-
  答案:46.college 47.various activities 48.communication 49.full-time 50.Student Advising Office
  Task 4
  Directions: The following is a list of medical terms. After reading it, you are required to find the items equivalent to (与…等同) those given in Chinese in the table below. Then you should put the corresponding letters in the brackets on the Answer Sheet, numbered 51 through 55.
  A—physical check-up
  B—biological clock
  C—pulse rate
  D—high blood pressure
  E—sleep habits
  F—over-the-counter (OTC) medicine
  G—side effect
  H—personality test
  I—operating room
  J—sense of well-being
  K—face-lift
  L—compulsive over-eating
  M—stress self-test
  N—plastic surgeon
  O—heart beat
  P—behavior therapy
  Q—terminal stage
  Examples: ( K ) 面部整容 ( N ) 整形外科医生
  51. ( ) 心跳 ( ) 非处方药
  52. ( ) 手术室 ( ) 强迫性过度饮食
  53. ( ) 副作用 ( ) 生物钟
  54. ( ) 高血压 ( ) 脉搏率
  55. ( ) 睡眠习惯 ( ) 体检
  答案:51.O,F 52.I,L 53.G,B 54.D,C 55.E,A
  Task 5
  Directions: The following is a business letter. After reading it, you should give brief answers to the 5 questions (No.56 to No.60) that follow. The answers (in no more than 3 words) should be written after the corresponding numbers on the Answer Sheet.
  Gmail (Google Email) Notice
  Subject: Your Gmail account has been inactive for a long time
  From “The Gmail Team” gmail-noreply@gmail.com
  Time 2008-06-26 14:12:55
  To wanghaiyan@163.com
  Hi there,
  We’ve noticed that you haven’t used your Gmail account, wanghaiyan@gmail.com, for quite some time. In order to make Gmail better for our users, we’ve added a lot of things in the last few months and we hope you’ll want to start using your account again.
  Stop worrying about storage (存储量). has 5000 megabytes (兆字节) of free storage and our plan is to continue growing your storage by giving you more space as we are able.
  The Gmail interface (界面) is now available in 12 languages. If you don’t see the language you want on this list, look for it in the future because we’re going to keep adding more.
  We’re still working hard every day to build for you the best email service around. But to keep Gmail great for our users, we may have to close inactive accounts after 9 months. So, we hope you’ll give us another chance. To log in (登录) to your Gmail account, just visit: http://gmail.google.com.
  Thanks.
  The Gmail Team
  56. Why does the Gmail Team write to the Gmail user?
  Because the user hasn’t used his / her Gmail ________ for quite some time.
  57. What kind of storage the user’s account now has?
  It now has 5,000 megabytes of _____________________________ storage.
  58. How many languages are available in the Gmail interface now?
  ___________________________________________________________.
  59. What may happen to the user’s account if it is inactive for 9 months?
  The Gmail team may have to _________________ the user’s account.
  60. What is the purpose of the Gmail team in sending this email?
  They hope the user will ___________________________ the account.
  答案:56.account 57.free 58.12 languages 59.close 60.log in

Part IV Translation — English into Chinese
  Directions: This part, numbered 61 through 65, is to test your ability to translate English into Chinese. After each of the sentences numbered 61 to 64, you will read four choices of suggested translation. You should choose the best translation and mark the corresponding letter on your Answer Sheet. And for the paragraph numbered 65, write your translation in the corresponding space on the Translation/ Composition Sheet.
  61. , training and personal qualities meet the requirements you set for the position of electrical engineer.
  A) 我认为我受的教育和培训,以及个人素质正是你们要求的,我会努力履行电气工程师的职务。
  B) 我相信我所接受的教育和培训,以及我的个人素质均符合你们对电气工程师职务设定的要求。
  C) 得知你们招聘电气工程师,我觉得我的背景可以让你们了解我受过的教育和培训,以及我的个人品德。
  D) 得知你们招聘电气工程师,和培训,完善我的个人品质,成为一名电气工程师。
  62. As you have done the market survey, I would like to discuss with you the possibility of selling our products in the US.
  A) 因为美国产品有市场,所以我们要研究开发我们产品的可能性。
  B) 因为你有市场资料,所以我希望与你讨论美国产品的销售情况。
  C) 你已经作了市场调查,所以我想和你讨论在美国销售我们产品的可能性。
  D) 你已了解了市场,因此我希望能与你商讨在美国开发我们产品的可能性。
  63. The general manager public relations person as well as the head of the company’s management.
  A) 总经理不仅要当好公关人员,而且要领导好公司。
  B) ,也一定要当好公司的领导。
  C) 总经理不仅是公司的管理首脑,还必须起某种公关人物的作用。
  D) 总经理要管理好自己的公司,首先必须管理好公司的公关人员。
  64. The study shows that our computers are superior to those of our competitors in terms of functions and speed.
  A) 研究表明,我们的计算机在功能和速度两方面都优于我们的竞争对手。
  B) 研究表明,我们的计算机与我们竞争者的产品在功能和速度方面有差异。
  C) 研究表明,我们的计算机在效率和速度方面都与其它厂商不同。
  D) 研究表明,我们的计算机正面临着高速发展的其它厂商的竞争。
  65. Movie-goers (电影院常客) across China will now only pay half price for tickets Tuesday, which will start in July. Any movie shown on Tuesday will be half price, even for new movies. The purpose is to bring people back to the cinema on Tuesday, when numbers of audiences are the lowest of the week. Some cinemas in Beijing and Shenzhen that began in advance have seen an increase in audiences Tuesday evenings.
  答案:61-64:BCCA         65.从七月开始,中国的电影院常客们每周二花半价就可以去电影院看电影。任何在周二上映的电影,即使是新电影也是半价。周二是一周中观众最少的一天,在这一天实行半价政策,是为了将人们带到影院。这项措施已经在北京和,这些影院周二晚上的观众数量都有所增加。
 2008年6月英语三级考试A级真题及答案

Part II Vocabulary & Structure
  Directions: This part is to test your ability to construct grammatically correct sentences. It consists of 2 sections.
  Section A
  Directions: In this section, there are 10 incomplete sentences. You are required to complete each one by deciding on the most appropriate word or words from the 4 choices marked A), B), C) and D). Then you should mark the corresponding letter on the Answer Sheet with a single line through the center.
  16. Most of the retired people are happy ________ their quiet life in the country- side.
  A) to
  B) of
  C) with
  D) on
  17. My brother reference books, but ________ of them was of any use for my report.
  A) neither
  B) none
  C) either
  D) all
  18. I don’t regret ________ her what I thought about her proposal, even if it upset her.
  A) tell
  B) to tell
  C) told
  D) telling
  19. ________ that I wasn’t going to get much chance for promotion, I soon became bored with my work.
  A) To realize
  B) Realizing
  C) Being realized
  D) Realized
  20. Please note that Boston next week, ________ you want to call me and discuss things.
  A) in case
  B) unless
  C) until
  D) so that
  21. I tried to get out of the business ________ I found impossible to carry on.
  A) why
  B) which
  C) what
  D) where
  22. Jack would rather spend time complaining than ________ the problem by himself.
  A) solve
  B) solved
  C) solves
  D) to solve
  23. They will not start the project until the board chairman ________ back from South Africa.
  A) will come
  B) is coming
  C) came
  D) comes
  24. Scientists should be kept ________ of the latest developments in their research areas.
  A) inform
  B) informing
  C) informed
  D) to inform
  25. Only when we had finished all the work ________ that it was too late to take a bus home.
  A) did we realize
  B will we realize
  C) we did realize
  D) we will realize
  Section B
  Directions: There are 10 incomplete statements here. You should fill in each blank with the proper form of the word given in brackets. Write the word or words in the corresponding space on the Answer Sheet.
  26. Obviously, nuclear power can never be the only (solve) ________ to energy crisis.
  27. It was in his childhood that he read most of the books (write) ________ by Mark Twain.
  28. Nobody at the meeting would (belief) ________ that the new proposal could be carried out smoothly.
  29.If the rent is as much as $750 a month, water, gas and electricity should (include) ________.
  30. The lecture was so (bore)________ that many classroom fell asleep.
  31. Mr. Smith considered (sell) ________ his car and his house before moving to Beijing.
  32. My mother (enjoy) ________ a better health since we came to live in this beautiful seaside city.
  33. The government is trying to find a way to deal with the problem of pollution (effective) ________.
  34. The young man did not have enough money; otherwise he (buy) ________ a more expensive watch.
  35. With the help of the police, the woman finally found her (lose) ________ child after a sleepless night.
  答案:16-20:CBDBA    21-25:BADCA
  26.solution 27.written 28.believe 29.be included 30.boring
  31.selling 32.has enjoyed 33.effectively 34.would buy 35.lost

 Part III Reading Comprehension
  Directions: This part is to test your reading ability. There are 5 tasks for you to fulfill. You should read the reading materials carefully and do the tasks as you are instructed.
  Task 1
  Directions: After reading the following passage, you will find 5 questions or unfinished statements, numbered 36 to 40. For each question or statement there are 4 choices marked A), B), C) and D). You should make the correct choice and mark the corresponding letter on the Answer Sheet with a single line through the center.
  It is the vision (设想) of the Public Transit Police Department to achieve a transit system free from crime and disorder. are designed to enhance safety, increase riders and preserve the quality of our system’s structure.
  Transit has its own police force committed to the safety of its customers and drivers, serving seven counties and 85 cities in the region. There are 23 full-time officers, 146 part-time officers and five administrative (行政的) staff, devoted to one thing: public safety.
  You’ve probably seen Transit Police officers patrolling(巡逻)bus routes in cars or on foot. During visits with drivers, Transit Police either ride along or they step on board to greet drivers and passengers. Sometimes officers patrol out of uniform – one could be on your next bus. The fare inspectors on Hiawatha light-rail trains areTransit Police officers.
  Every new Public Transit bus has an onboard security camera. Videotapes from these cameras help Transit Police identify and charge criminals.
  Downtown patrols built up in early 2002 have improved the quality of life for downtown residents and transit customers. Ten full-time police officers are assigned to the Minneapolis and St. Paul downtown areas, to ensure public security.
  Downtown arrests have gone from 490 in April-September 2001 to 941 during the same period in 2002.
  36. The responsibility of the police force of Public Transit is to ________.
  A) enhance public security
  B) build a free transit system
  C) control the number of riders
  D) ensure a convenient transit system
  37. Sometimes patrolling officers on because ________.
  A) they look like fare inspectors
  B) they don’t wear uniform
  C) they are polite to passengers
  D) they often take a back seat
  38. The purpose of installing a camera inside the new buses is to ________.
  A) keep a close watch over the road ahead
  B) take pictures of the passengers on board
  C) increase the efficiency of the transit system
  D) help the transit police detect criminals
  39. Two figures are mentioned in the last paragraph to indicate ________.
  A) the Public Transit system’s efficiency
  B) the success of Transit Police program
  C) the police’s emphasis on security of downtown areas
  D) the worsening security condition of the area
  40. The best title for the passage might be ________.
  A) Police and Security
  B) New Security Measures
  C) Transit Police Security Program
  D) Crime-free Public Transit System
  Task 2
  Directions: This task is the same as Task 1. The 5 questions or unfinished statements are numbered 41 to 45.
  The White House is the most visited residence in the world. Tours may be scheduled through our Washington DC office. Due to security and scheduling procedures, there are a few things you should know before requesting a tour:
  • Only groups of 10 or more may request a tour.
  • Requests must be submitted with a minimum of one month’s advance notice from the date of the tour. Notice of whether your application is accepted will be given 10 days prior to the date requested. All tours, even after they have been confirmed, are subject to cancellation due to security interests.
  • Tours are only conducted Tuesday through Saturday from 7:30 am to 11:30 am.
  To process your request for a tour, please contact our Washington office at (202) 224-5521 and provide the following information:
  • Date(s) requesting.
  • Security information for each person in your party, including: name as it appears on I.D. (I.D. required for ages 14 and up), date of birth, social security number, country of origin, and citizenship (公民的身份).
  • A home address and daytime and evening phone numbers for the designated (指派的) leader of the group.
  • A contact number while in DC for the designated leader of the group.
  After your request is , our office will contact you with further instructions.
  41. This notice is to provide information about ________.
  A) scheduling of Washington DC tours
  B) the security system of the White House
  C) application for a tour of the White House
  D) duties of a tour group leader
  42. The White House is open to the public ________.
  A) on weekdays only
  B) five days a week
  C) every other day
  D) on weekends only
  43. Application for a group tour of the White House must be submitted ________.
  A) exactly on the date of application
  B) at least 30 days in advance
  C) any day between Tuesday and Saturday
  D) 10 days before the date of the tour
  44. After your tour request has been confirmed, ________.
  A) it is still possible that your tour may be cancelled
  B) you are still allowed to change your visiting date
  C) you can surely visit the White House that day
  D) it is necessary for you to start your tour immediately
  45. It can be inferred from the passage that ________.
  A) teenagers under 14 are not allowed to visit the White House
  B) foreigners are less likely to be permitted to tour the White House
  C) separate tours can also be scheduled for individual visitors
  D) security is the chief concern in scheduling White House tours
  答案:36-40:DBDBC      41-45:CBBAD

Task 3
  Directions: The following is a letter of application. After reading it, you are required to complete the outline below it (No.46 to No.50). You should write your answers briefly (in no more than three words) on the Answer Sheet correspondingly.
  Dear Mr. Williams:
  Your advertisement in for manager of public relations appeals to me. I found the wording of your advertisement quite attractive with emphasis on leadership, initiative, and flexibility. And my experience and qualifications indicate that I am the person you are seeking.
  The enclosed résumé indicates my experience in the area of public relations and management communications. I am quite familiar with the kinds of issues and problems that you have to deal with.
  I’d like to draw your attention to page 2 of my résumé, on which I describe my concept of public relations. And I am most eager to put this concept into practice to prove it to you.
  Although I have been very happy with my present employer and colleagues, I am more willing to join your company where I can assume even broader responsibility.
  I am free to travel and open to relocation. I would welcome the opportunity to meet you and to further discuss how I may benefit your organization. Please call me at 0411-89726374 to arrange an interview at your earliest convenience.
  Sincerely yours,
  Stephen Smith
  An Application Letter
  Applicant: -46-
  Position applied for: the manager of -47-
  Requirements emphasized in the ad: 1. leadership
  2. initiative
  3. -48-
  Expectation of the applicant: to assume -49-
  Contact telephone number: 0411-89726374
  Purpose of the letter: asking for -50-
  答案:46.Stephen Smith 47.public relations 48.flexibility 49.broader responsibility 50.an interview
  Task 4
  Directions: The following is a list of terms related to parcel delivery service. After reading it, you are required to find the items equivalent to (与…等同) those given in Chinese in the table below. Then you should put the corresponding letters in the brackets on the Answer Sheet, numbered 51 through 55.
  A—parcel carrier
  B—annual revenue
  C—package distribution company
  D— quality
  E—weight of the shipment
  F—pick-up and delivery
  G—urgent delivery
  H—full electronic tracking
  I—export documentation
  J—scheduled delivery
  K—automatic confirmation of delivery
  L—global network
  M—door-to-door delivery
  N—zone number of the destination
  O—approximate value of the shipment
  P—additional charge
  Q—express plus package
  Examples: (Q) 加急邮件服务 ( F ) 收取与递送服务
  51. ( ) 定时送货 ( ) 紧急投递
  52. ( ) 全程电子跟踪 ( ) 货物大约价值
  53. ( ) 包裹承运商 ( ) 自动送货确认
  54. ( ) 质量承诺 ( ) 允许出口的文件
  55. ( ) 附加费 ( ) 上门送货服务
  答案:51.J,G 52.H,O 53.C,K 54.D,I 55.P,M
  Task 5
  Directions: The following is a Call for Partnership(征求合作伙伴)advertised in a newspaper. After reading it, you should give brief answers to the 5 questions (No.56 to No.60) that follow. The answers (in no more than 3 words) should be written after the corresponding numbers on the Answer Sheet.
  In order to hold an outstanding Olympic Games, the Beijing Organizing Committee issues this Call for Expression of Interest in the project of Beijing 2008 Partner. Interested parties are invited to participate in the procedure and submit their declaration of interest and related supporting documents.
  General Terms and Conditions of Participation
  1. Applicants must be able to contribute sufficient funds, products and related services to meet the demands of the Beijing 2008 Olympic Games.
  2. Applicants must possess , financial and technical capabilities.
  3. Applicants must enjoy a positive social image, good reputation and excellent business record.
  4. Applicants undertake not to associate their response to this call and participation in the procedure with any of their commercial activities.
  56. What is the main purpose of this Call for Expression of Interest?
  To invite _____________ to participate in the Beijing 2008 Olympic Games.
  57. Who is the issuer of this Call for Expression of Interest?
  The ____________________________________ of the 2008 Olympic Games.
  58. What must the applicants do to meet the demands of the 2008 Olympic Games?
  Contribute sufficient funds, products, and ___________________________.
  59. What is required of the applicants in terms of social image?
  They must enjoy a ___________________________________ social image.
  60. What should applicants undertake not to do?
  Associate their response and participation with any of their __________.
  答案:56.intrested parties 57.Beijing Organizing Committee 58.related services
  59.positive 60.commercial activities

Part IV Translation — English into Chinese
  Directions: This part, numbered 61 through 65, is to test your ability to translate English into Chinese. After each of the sentences numbered 61 to 64, you will read four choices of suggested translation. You should choose the best translation and mark the corresponding letter on your Answer Sheet. And for the paragraph numbered 65, write your translation in the corresponding space on the Translation/ Composition Sheet.
  61. us of the overcharge on your account and we have contacted the store on your behalf and are awaiting their reply.
  A) 承蒙告知您受到恶意透支的指控,我们已经派代表与商店联系并正在等待回音。
  B) 承蒙告知您的帐户存在问题,我们已经和商店联系过并正在等待他们的回答。
  C) 承蒙告知您的帐户被多扣款一事,我们已代您与商店联系,正在等待他们回复。
  D) 承蒙告知有人指控您恶意透支,我们代表商店向您致歉,请您等待他们的答复。
  62. There is no way we’ll get lost in the mountains, since the tour guide has figured out the return route.
  A) 我们根本不会在山里迷路,因为导游已回到了原来的路线上。
  B) 既然导游已经弄清了返程的路线,我们就绝不会在山里迷路。
  C) 因为我们在山里迷失了方向无路可走,导游只好按原路返回。
  D) ,所以导游一直都在寻找返回的路线。
  63. With all its disadvantages, the new design they have submitted should still be considered as one of the best at present.
  A) 尽管他们提交的这个新设计有种种不足,但仍应视为目前最佳设计之一。
  B) 尽管有许多不利因素,他们做出的新设计仍然被视为目前最佳设计之一。
  C) 如果没有这些缺陷,他们的新设计就应该被视为迄今为止最佳的一种设计。
  D) 如果没有这么多不利因素,他们的新设计就应该作为目前最佳的一个方案。
  64. If you require , your application should be in the Student Financial Aid Services at least three weeks before your registration date.
  A) 如果你需要资助学费,应向助学办公室提出申请,。
  B) 如果你需要申请贷款交纳学费,你应该至少在三周内到助学办公室登记注册。
  C) 如果你需要减免学费,应到助学办公室提出申请,手续。
  D) 如果你需要学费资助,最迟应该在注册日期三周前将你的申请交到助学办公室。
  65. The sales service includes installation (安装) and training within one week of receiving the order. On the day of installation, our training engineer will your secretary, providing instructions on the operation of this easy-to-use photocopier (复印机). You may call us for help at any time in the future. If necessary, our trainer will go to your company to help you with the operation of the machine.
  答案:61-64:CBAD    65:订单一周内的培训。在安装机器当天,我们的工程师将培训您的秘书一个小时,讲解便捷的复印机操作规范。您可以在任何时间拨打我们的帮助热线。如有必要,我们的培训师将前往贵公司帮助您操作机器。
  

2007年12月英语三级考试A级真题及答案

Part II Vocabulary & Structure
  Directions: This part is to test your ability to construct grammatically correct sentences. It consists of 2 sections.
  Section A
  Directions: In this section, there are 10 incomplete sentences. You are required to complete each one by deciding on the most appropriate word or words from the 4 choices marked A), B), C) and D). Then you should mark the corresponding letter on the Answer Sheet with a single line through the center.
  16. Although ,he insisted _________going there together with us.
  A) to
  B) on
  C) at
  D) for
  17. I’ll ask Mr. Smith to ring you up _________he comes back to the office.
  A) when
  B) where
  C) because
  D) although
  18. They regard _________as their duty to provide the best service for their customers.
  A) this
  B) what
  C) it
  D) that
  19. Not until the day before yesterday _________to give a speech at the meeting.
  A) he agreed
  B) does he agree
  C) he agrees
  D) did he agree
  20. _________up at the clock on the wall,the secretary found it was already midnight.
  A) Looking
  B) Look
  C) To look
  D) Looked
  21. The first textbook _________for teaching English as a foreign language came out in the 16th century.
  A) writing
  B) written
  C) to write
  D) to be written
  22. Young _________he is, an able salesman.
  A) that
  B) who
  C) as
  D) which
  23. I don’t doubt _________the stock market will recover from the economic crisis.
  A) if
  B) what
  C) that
  D) which
  24. In our company,great changes _________since the new manager came.
  A) took place
  B) take place
  C) will have taken place
  D) have taken place
  25. News came from the sales manager _________the new product had been selling well in the local market for three months.
  A) whose
  B) what
  C) which
  D) that
  Section B
  Directions: There are 10 incomplete statements here. You should fill in each blank with the proper form of the word given in brackets. Write the word or words in the corresponding space on the Answer Sheet.
  26. Sandy made quite a number of(apply) _________for a management position but failed every time.
  27. The graduates had a(cheer) _________farewell party before leaving the college.
  28. Karl probably(see) _________“Star Wars” several times for he knows every detail of the film.
  29. Believe it or not,when first(introduce) _________to Europe,tomato was thought to be poisonous.
  30. A telephone user may pay a fixed monthly charge and(allow) _________to make an unlimited number of local calls in the month.
  31. I remember(see) _________you somewhere before,but I can’t tell the exact place.
  32. I don’t think it necessary(discuss) _________the the problem is settled.
  33. Since five managers are going to give their reports,the meeting(last) _________for at least two hours.
  34. Although he was(deep) _________hurt by what she said to him,he made no reply.
  35. It is strongly recommended that teachers(use) _________computers to assist in their classroom teaching.
  答案:16-20:BACDA         21-25:BCCDD
  26.applications 27.cheerful 28.has seen 29.introduced 30.be allowed
  31.having seen 32.to discuss 33.will last 34.deeply 35.use/should use

 Part III Reading Comprehension
  Directions: This part is to test your reading ability. There are 5 tasks for you to fulfill. You should read the reading materials carefully and do the tasks as you are instructed.
  Task 1
  Directions: After reading the following passage, you will find 5 questions or unfinished statements, numbered 36 to 40. For each question or statement there are 4 choices marked A), B), C) and D). You should make the correct choice and mark the corresponding letter on the Answer Sheet with a single line through the center.
  Melbourne,with a population of over 3.5 million,is the second largest city in Australia. It is clean,safe,dynamic and exciting,and well known internationally for its universities and other educational institutions.
  The city has well-planned tree-lined beautiful parks and gardens. It has a good transport system of roads,buses,trains,and trams(电车). The La Trobe University(拉特罗布大学) campus is connected to the Central Business District by trams,express buses,and bus and train connections.
  Melbourne is a culturally rich city,and is home to large communities of people from all parts of Europe,the Americas,Africa,and Asia. The city is famous for its restaurants,theatres,music,opera,ballet,art,culture,and shops,and a lively and dynamic nightlife. Melbourne people are enthusiastic about sports,and the city hosts many famous international sports events. Near Melbourne there are beautiful coastlines with excellent beaches,national parks,forests,wineries(葡萄酒厂) ,winter snowfields and summer resorts.
  The climate is temperate and comfortable,with warm summers and cool winters. In summer,maximum daytime temperatures range from 26℃to 36℃,and in winter from 12℃ to 18℃.1]he weather in
  Melbourne can be variable from day to day. In 2002.Melbourne was rated the world’s best city to live in by the Economist Intelligence Unit.
  36. Melbourne is well-known in the world for its__________________.
  A) large population
  B) educational institutions
  C) transport system
  D) beautiful parks and gardens
  37.According to the passage,Melbourne is a city where_________.
  A) rich people choose to live
  B) the best wine is produced
  C) various cultures exist
  D) Asian food is popular
  38. Melbourne people are very interested in_________.
  A) sports
  B) sunbathing
  C) sightseeing
  D) traveling
  39. The word temperate in the sentence “The climate is temperate…”(Line 10) ,most probably means ________.
  A) hot
  B) mild
  C) dry
  D) cold
  40. The best title for the passage might be_________.
  A) An Ideal Place for Shopping
  B) A City with the Best Climate
  C) The World’s Best City to Live in
  D) The World’s Most Beautiful City
  Task 2
  Directions: This task is the same as Task 1. The 5 questions or unfinished statements are numbered 41 to 45.
  Terms and Conditions of Employment
  These terms and conditions should be read before you sign your contract.
  Salary:Your starting salary is:$15,000.This is reviewed annually.
  Hours: work are eight hours a day,Monday through Friday. You will start work on Feb.21,2008.On the first morning,report to your line manager John Knight.
  Health and safety:Please read the safety regulations attached. If you have any questions,contact the health and safety officer,whose name is at the top of the regulation sheet. If you have health problems,please inform the Senior Nurse,Chris Thomas. If you cannot work because of illness,please telephone the factory.
  Annual leave:During your first year of employment you ale allowed twenty days’ leave. This should be arranged with your line manager.
  Overtime(加班) :If you work more than forty hours a week,you will be paid at the current overtime rate. Your line manager will keep a record of the overtime you work. If you work on public holidays,you will be paid at the current rates. If you prefer,time can be taken instead of extra pay for public holidays and overtime.
  Clothing:The Supplies Department provides overalls(工作服) .Inform Supplies of your size two days before you need them. You can also order any other equipment you need for your job from Supplies.
  41. According to the contract,the salary of the employee_________.
  A) is paid weekly
  B) is adjusted every quarter
  C) is re-examined from year to year
  D) is fixed for the whole contract period
  42. If employees have any health problem,they should_________.
  A) telephone the factory
  B) inform the Senior Nurse
  C) report to t11eir line manager
  D) contact the health and safety officer
  43. The line manager is responsible for____________.
  A) giving a pay raise
  B) signing a contract
  C) arranging annual leave
  D) helping with personal affairs
  44. If employees work on public holidays,usually they will get_________.
  A) extra days off
  B) extra money
  C) a pay raise
  D) paid holidays
  45. When they need overalls for their job,employees should inform the Supplies Department of_________.
  A) the size
  B) the color
  C) the style
  D) the quantity
  答案:36-40:BCABC      41-45:CBCBA

Task 3
  Directions: The following is part of a job advertisement. After reading it, you are required to complete the outline below it (No.46 to No.50). You should write your answers briefly (in no more than three words) on the Answer Sheet correspondingly.
  INFORMATION SECURITY SPECIALIST
  NOTE:Please directly through this website,as well as emailing your resume as an attachment to us at resumes@ictva.com
  JOB DESCRIPTION:
  Establishes and satisfies highly challenging and complex information system security(ISS) requirements based upon the analysis of user,operational,policy,regulatory,and resource demands.Supports the development,implementation,and operation of ISS-enabling technologies, processes, and procedures into client systems and networks.Also requires an expert understanding of security policy advocated by the U.S Government including Department of Defense and other agencies,e.g.,Treasury,as well as commercial best security practices.
  EDUCATION:
  Bachelor’s degree in Computer Science or related fields.
  REQUIRED SKILLS:
  ISS Policy,Procedures,and Planning,RACF,LAN,LIFE CYCLE,Microsoft NT,Network Security,Office Automation Tools.
  LOCATION:
  New Carrollton,Maryland or Falls Church, Virginia
  CONTACT US AT:
  resumes@icta.com
  U.S. Citizenship or U.S. Permanent Resident status required for employment.
  A job advertisement
  Position offered: -46-
  Responsibilities:to establish and satisfy -47- requirements
  Qualifications:1. -48- degree
  2.required skills
  Citizenship required:US Citizenship or -49-
  Way of contact:by -50-
  答案:46.information security specialist 47.information system security 48.bachelor’s
  49.Permanet Resident status 50.e-mail
  Task 4
  Directions: The following is a list of terms for library signs. After reading it, you are required to find the items equivalent to (与…等同) those given in Chinese in the table below. Then you should put the corresponding letters in the brackets on the Answer Sheet, numbered 51 through 55.
  A—reading,reference and copying
  B—staff only
  C—closed shelves for undergraduates
  D—information retrieval room
  E—inter-library loan
  F—display&reading room for new books
  G—multi-media reading room
  H—reference department office
  I—novelty research
  J—lecture hall
  K—reading room for reference books
  L—return deposit
  M—reading room
  N—title catalogue
  O—circulation for foreign books
  P—card catalogue
  Q—periodicals and magazines
  Examples: (O) 外文书刊借阅 (D) 信息检索室51.( ) 员工专用 ( ) 多媒体阅览室
  52.( ) 参考书阅览室 ( ) 中文社科图书阅览室
  53.( ) 报告厅 ( ) 书名目录
  54.( ) 卡片目录 ( ) 新书展阅室
  55.( ) 期刊杂志阅览 ( ) 馆际互借
  答案:51.B,G 52.K,M 53.J,N 54.P,F 55.Q,E
  Task 5
  Directions: The following is an agreement. After reading it, you should give brief answers to the 5 questions (No.56 to No.60) that follow. The answers (in no more than 3 words) should be written after the corresponding numbers on the Answer Sheet.
  (Website) Visitor Agreement
  Statesman.com is provided to you by Cox Texas Newspapers. This visitor agreement is legally binding(有约束力的) between you and us. Please read this visitor agreement;by using this service,you accept its terms. The Internet is a fast growing medium;we may change the terms of this agreement from time to time. By continuing to use the service after we post any such changes,you accept this agreement as modified.
  We reserve the fight to deny access to this website,or any service provided via this website,to anyone who violates(违反) this visitor agreement or who,in our judgment,interferes with the ability of others to enjoy this website,or infringes(侵犯) the fights of others.
  We invite you to send in your questions or comments about this website,or to any material you believe to be inaccurate. Please send such comments,including a copy of any material you wish to discuss to:
  Jim Smith
  General Manager
  Statesman.com
  305 South Congress Avenue
  Austin,Texas 78704
  Phone:(512) 912—2510
  Fax:(512) 912-2926 Or e—mail us.
  56. Who are the two parties to the agreement?
  The provider of the website and its __________________.
  57. What is meant if you go on using the service after changes in the agreement are posted?
  It means you have __________________the modification.
  58. What will happen if a visitor breaks this agreement?
  The visitor will be denied access to this__________________.
  59. What are visitors invited to do about this website?
  Send in their __________________about the website.
  60. What can visitors do when they find materials on the website inaccurate?
  They can send their comments to__________________.
  答案:56.visitors 57.accepted 58.website 59.questions or comments 60.Jim Smith/the general manager

Part IV Translation — English into Chinese
  Directions: This part, numbered 61 through 65, is to test your ability to translate English into Chinese. After each of the sentences numbered 61 to 64, you will read four choices of suggested translation. You should choose the best translation and mark the corresponding letter on your Answer Sheet. And for the paragraph numbered 65, write your translation in the corresponding space on the Translation/ Composition Sheet.
  61. professional hair—care products indicates that consumers are paying as much attention to their hair as to their skin.
  A) 专业护发产品的增加表明消费者既关心护发,也关心护肤。
  B) 专业护发产品销量的增加表明消费者既重视护肤,也重视护发。
  C) 随着美发行业的发展,消费者越来越重视美发和护肤。
  D) 随着美发行业的发展,消费者认为美发和护肤同等重要。
  62. Women would double their risk of suffering from lung cancer if they were exposed to 40 or more years of household tobacco smoke.
  A) 妇女在家里吸烟长达410年或更长时间,她们患肺癌的痛苦就会加倍。
  B) 家庭妇女接触油烟,她们患肺癌的痛苦就会加倍。
  C) 妇女在家里吸烟的时间,她们患肺癌的危险就会加倍。
  D) 妇女在吸烟的家庭环境中生活40年或更长时间,她们患肺癌的风险就会加倍。
  63. Though technically quite advanced today,the Internet is far from being popular with average household users in some developing countries.
  A) 尽管互联网技术现在已经很先进,但在一些发展中国家还未受到家庭用户的广泛青睐。
  B) 很先进,但在一些发展中国家还远远没有普及到一般家庭。
  C) 尽管今天科学技术已经很先进,但在一些发展中国家互联网离大众的生活还是很远。
  D) 尽管今天科学技术已经很先进,但在一些发展中国家互联网还没有进人普通家庭。
  64. Your have sold SO well but for a lot of advertisements we put on the television.
  A) 如果我们没在电视上作大量广告,你们的新产品就不会有很好的销路。
  B) 虽然我们未在电视上作大量广告,你们的产品依然销路很好。
  C) 若不是我们,你们的新产品销路不会这么好。
  D) 尽管我们在电视上作了大量的广告,你们的产品依然销售不好。
  65. The regulations make it safer for you to use your credit card for shopping on the Internet or over the phone. If someone else makes dishonest use of your credit card。you Can cancel the payment and the card issuer(发卡机构) must refund the money to your account. The card issuer will want you to tell them as soon as possible if your card has been lost or that someone has used your card details dishonestly.
  答案:61-64:BDBC        65:这些规章制度可以保证你在使用信用卡进行网上或者电话购物时更安全。如果别人盗用你的信用卡进行交易,你可以取消支付,发卡机构必须将钱退还到你的账户。党你的信用卡丢失或被盗或是你发现有人盗用了你的信用卡时,发卡机构希望你能及时告知他们。
  2007年6月英语三级考试A级真题及答案

Part II Vocabulary & Structure
  Directions: This part is to test your ability to construct grammatically correct sentences. It consists of 2 sections.
  Section A
  Directions: In this section, there are 10 incomplete sentences. You are required to complete each one by deciding on the most appropriate word or words from the 4 choices marked A), B), C) and D). Then you should mark the corresponding letter on the Answer Sheet with a single line through the center.
  16. There are two maps on the wall:one is a map of China,and ___________ is a map of the world.
  A) other
  B) another
  C) the other
  D) the others
  17. For years,doctors ___________millions of patients’ lives with .
  A) have saved
  B) ale saving
  C) will save
  D) were saving
  18. Once more I have to leave Beijing,___________I have been living for eight years.
  A) that
  B) where
  C) which
  D) as
  19. I was almost asleep last night when I suddenly heard someone _____at the door.
  A) be knocking
  B) knocking
  C) to knock
  D) having knocked
  20. The conference ___________in Beijing next week is bound to be a great success.
  A) holding
  B) being held
  C) to hold
  D) to be held
  21. It wasn’t such a good job ___________she had read about in the advertisement.
  A) like
  B) which
  C) as
  D) what
  22. It’s my great honor ___________to give a speech at the opening ceremony.
  A) to invite
  B) inviting
  C) having invited
  D) to be invited
  23. Not until yesterday _______ project that will be completed soon.
  A) did I learn
  B) have I learnt
  C) I learnt
  D) that I learnt
  24. This problem is ___________his ability and I don’t think he Call solve it.
  A) to
  B) in
  C) beyond
  D) under
  25. She didn’t go to the party last night,___________she had to finish her term paper.
  A) if
  B) though
  C) till
  D) because
  Section B
  Directions: There are 10 incomplete statements here. You should fill in each blank with the proper form of the word given in brackets. Write the word or words in the corresponding space on the Answer Sheet.
  26. The engineers spent the whole night(work) ___________on the new device.
  27. I’m not sure whether we can gain any profit from the(invest) ___________.
  28. The price of petrol is much(high) ___________now than it was this time last year.
  29. No one can deny that we(make) ______tremendous progress in the past twenty years.
  30. The research group has submitted a report,(suggest) ______reforms to be made.
  31. The people injured in the accident(send) ___________to the nearest hospital for treatment last night.
  32. The organization started a(nation) ___________campaign against cigarette smoking in public places.
  33. Application for this training course should be sent(direct) ___________to the admission office.
  34. the manager(pay) ___________more attention to the services for the customers.
  35. When(ask) ___________about the advertising campaign of the new product,the manager said it was a great success.
  答案:16-20:CABBD    21-25:CDACD
  26.working 27.investment 28.higher 29.have made 30.suggesting
  31.were sent 32.national 33.directly 34.paid 35.asked

Part III Reading Comprehension
  Directions: This part is to test your reading ability. There are 5 tasks for you to fulfill. You should read the reading materials carefully and do the tasks as you are instructed.
  Task 1
  Directions: After reading the following passage, you will find 5 questions or unfinished statements, numbered 36 to 40. For each question or statement there are 4 choices marked A), B), C) and D). You should make the correct choice and mark the corresponding letter on the Answer Sheet with a single line through the center.
  Everybody telecommuting(远程办公) .“It won’t work in most jobs”,“It costs too much”,“It reduces air pollution”,“It helps people balance family and work responsibilities”,and “Most people are doing it”.
  In reality researchers continue to find strong growth and acceptance of telecommuting. Nearly two-thirds of the top 1000 companies in the world have a telecommuting program,and 92 percent say it reduces cost and improves worker productivity(生产力) .The days of everyone commuting to the office five days a week are quickly disappearing.
  Telecommuting involves a non-traditional work arrangement enabling workers to work at home or elsewhere,some or all of the time. This is not a new,novel,or untested way of working.
  But is it for you? Telecommuting is not a panacea(万能药) .Whether you are a manager,or an HR(Human Resources) specialist,there are decisions to make and actions to take before you begin a telecommuting arrangement.
  Join us for any or all of the following meetings to get answers,information,and resources to develop and carry out a successful telecommuting arrangement. Each meeting offers you an informative presentation followed by the opportunity for a discussion with a panel of “experts” who have made telecommuting work for them.
  36. How do people look at telecommuting according to the first paragraph?
  A) They are against it.
  B) They don’t care about it.
  C) They share the same view.
  D) They differ in their opinions.
  37. According to the response of most of the top 1000 companies, telecommuting_________.
  A) increases worker productivity
  B) will disappear in the near future
  C) cannot be accepted by the public
  D) is practiced in all the top companies
  38. Which of the following statements is TRUE of telecommuting?
  A) It is up to the employees to accept it or not.
  B) It is getting popular in different companies.
  C) It is a new untested way of working.
  D) It is a traditional work arrangement.
  39. Before beginning a telecommuting arrangement,the management should______.
  A) appoint a new HR specialist
  B) provide the facilities and conditions
  C) improve the company’s productivity first
  D) decide whether it is suitable for the company
  40. According to the last paragraph,meetings are held to___________.
  A) appreciate the efforts of the telecommuting companies
  B) discuss the employment of telecommuting experts
  C) help introduce the practice of telecommuting
  D) train people before
  Task 2
  Directions: This task is the same as Task 1. The 5 questions or unfinished statements are numbered 41 to 45.
  Rockwatch—The Best Club on Earth
  If you are a young person and interested in geology(地质学) ,then Rockwatch is the club for you.
  When You Join
  New memberships receive a Rockwatch Rox file each. This has the information and top tips you will need to start enjoying geology. It’s designed to serve as your own field notebook as well.
  In it you will find your:
  membership card
  full color mini-map
  thumbs up guide
  fact cards
  Rockwatch Magazine
  Our lively magazine is mailed to members three times a year. They can read reports and news from around the world,and articles on everything from diamonds to dinosaurs(恐龙) ,earthquakes to erosion(水土流失) .
  Rockwatch Events
  With each magazine you will receive a Rockwatch events calendar. Rocky activities suitable for families are listed and include road shows and guided walks.
  The Rockwatch Rock Artist
  Are you an artist, or a photographer? This is your chance to become Rockwatch Rock Artist of the Year and win amazing prizes in Our annual competition.
  Special Offer
  Rockwatch members can have specially discounted Wildlife Watch membership. Watch is the biggest environmental action club for young people,with 100 groups across the country. You can join both clubs together in the membership form.
  41. New Rockwatch Rox Club members will obtain a special file when they___________.
  A) do field work
  B) join the club
  C) buy a field notebook
  D) start studying geology
  42. Rockwatch is a magazine telling about things related to____________.
  A) geology
  B) agriculture
  C) politics
  D) economics
  43. What activities are specially arranged for Rockwatch members interested in photography?
  A) Guided walks.
  B) Rocky activities.
  C) Yearly competitions.
  D) Academic workshops.
  44. When applying for Wildlife Watch membership,a Rockwatch member Can enjoy___________.
  A) free membership
  B) a special discount
  C) a Rock Artist prize
  D) guided road shows
  45. You may join both Rockwatch and Wildlife Watch clubs by___________.
  A) calling the two clubs
  B) providing references
  C) applying separately
  D) filling in one form
  答案:36-40:DABDC      41-45:BACBD

Task 3
  Directions: The following is a is part of an instruction. After reading it, you are required to complete the outline below it (No.46 to No.50). You should write your answers briefly (in no more than three words) on the Answer Sheet correspondingly.
  NUROFEN RECOVERY(纽洛芬去痛片)
  Please carefully before you take this medicine.
  Nurofen Recovery dissolves(溶解) quickly on the tongue without the need to use water. It delivers effect .
  You should not take Nurofen Recovery if:
  —you have had an allergic(过敏的) reaction to aspirin(阿司匹林)
  —you have had a worsening of asthma(哮喘) when taking aspirin or similar medicines
  —you are under 12 years of age
  Administration:
  Place a tablet on the tongue,allow it to dissolve and then swallow—now water is required。
  Adults, the elderly and children of 12 years and older:
  Take 2 tablets,then if necessary,take 1 or 2 tablets every 4 hours. Do not exceed 6 tablets in 24 hours. Not suitable for children under 12 years.
  Warnings:
  If you take too many tablets by mistake,contact your doctor as soon as possible.
  If symptoms persist or if new symptoms occur,consult your doctor.
  Possible side effects:
  Stomach discomfort or pain,worsening of asthma 01″shortness of breath. If you experience any of these, stop taking the tablets and see your doctor.
  NUROFEN RECOVERY
  Function:delivering -46- from headaches
  Feature:dissolving quickly on -47-
  People not intended for:1) patients with an allergic reaction to aspirin
  2) patients having asthma when taking aspirin
  3) children under the age of -48-
  Dosage:1) Starting Dose:2 tablets
  2) If necessary:take l or 2 tablets -49-
  3) Maximum daily intake:6 tablets
  Possible side effects:1) stomach discomfort
  2) worsening of asthma
  3) -50-
  答案:46.effective relief 47.the tongue 48.12 49.every 4 hour 50.shortness of breath
  Task 4
  Directions: The following is a list of airport items. After reading it, you are required to find the items equivalent to (与…等同) those given in Chinese in the table below. Then you should put the corresponding letters in the brackets on the Answer Sheet, numbered 51 through 55.
  A—Waiting and Boarding
  B—Luggage Delivery
  C—Inspection and Quarantine
  D—Getting a Boarding Pass
  E—Security Check
  F—Domestic Departure
  G—Over-sized Luggage Checked-in
  H— Hand-carried
  I—Duty-free Articles
  J—Customs Declaration Form
  K—Quantity Allowed to Take
  L—Regulations on Restriction of Liquids
  M—Temporary Boarding ID Card
  N—Guide to Outgoing Passengers
  O—Goods Prohibited to Exit the Country
  P—Restriction of Hand Carry-on Articles
  Q—Detection Passage
  Examples:(L) 限带液体物品的规定 (C) 检查与检疫
  51.( ) 离港旅客指南 ( ) 领取登机牌
  52.( ) 禁止携带出境的物品 ( ) 大件行李托运
  53.( ) 候机/登机 ( ) 禁止随身携带的物品
  54.( ) 限带物品数量 ( ) 检查通道
  55.( ) 海关申报表 ( ) 免税物品
  答案:51.N,D 52.O,G 53.A,H 54.K,Q 55.J,I
  Task 5
  Directions: The following is part of an introduction to life insurance. After reading it, you should give brief answers to the 5 questions (No.56 to No.60) that follow. The answers (in no more than 3 words) should be written after the corresponding numbers on the Answer Sheet.
  When you buy life insurance,you want a policy that fits your needs at reasonable cost. Your first step is to determine how much life insurance you need. Next,you need to decide how much money you can afford to pay. Finally,you must policy that meets your coverage(保险类别) goals and fits into your financial plan. Once you have completed these steps,you will be able to move ahead and contact several life insurance companies through an agent who will show the fight type of policy for you.
  There are many reasons for purchasing life insurance,among which are the following:
  Insurance to provide family protection and financial security to surviving family members upon the death of the insured person.
  Insurance to cover a particular need upon the insured’s death such as paying off a mortgage or other debts.
  56. What should you take into consideration when choosing a life insurance policy?
  Both your needs and the ______________________.
  57. What’s the relationship between the type of policy and your financial plan?
  The type of policy should meet your______________________.
  58. Who can help you buy the fight type of policy from an insurance company?
  ____________________.
  59. Who will benefit from the life insurance upon the death of the insured person?
  Surviving ______________________.
  60. What is the second goal for buying life insurance?
  To pay off a mortgage or ______________________ after death.
  答案:56.cost 57.coverage goals 58.An angent 59.family members 60.other debts

 Part IV Translation — English into Chinese
  Directions: This part, numbered 61 through 65, is to test your ability to translate English into Chinese. After each of the sentences numbered 61 to 64, you will read four choices of suggested translation. You should choose the best translation and mark the corresponding letter on your Answer Sheet. And for the paragraph numbered 65, write your translation in the corresponding space on the Translation/ Composition Sheet.
  61. If you are parent or know someone who is, this 30-page booklet may be of great help to you.
  A) 假如你注意的话,你会知道这本小册子有30页,会对你的父母和你认识的人很有用。
  B) 要是这本30页的小册子很有用,可以给你的父母和你认识的人各买一本。
  C) 如果你所照顾的父母或认识的亲人年事已高,这本30页的小册子会对你有很大的帮助。
  D) 若你在照看年迈父母或知道有人在这样做,这本30页的小册子会对你们有很大帮助。
  62. Bidding documents can be obtained at the following address upon receipt of a non-refundable payment of RMB ¥2.799 each set.
  A) 标书要寄到下列地址,2,799元,标书文本不得更改。
  B) 招标文件可在下列地址获取,每套文件为人民币2,799元,不可退换。
  C) 可向下列地址索取招标文件,每套需支付人民币2,799元,恕不退款。
  D) 到下列地点办理相关招标手续,交付人民币2,799元,标书不得更改。
  63. During the meeting held in Brazil last month the supporters of free economic policies could benefit all nations.
  A) 支持者上个月在巴西召开自由贸易会议,他们辩称这些经济政策有利于所有的国家。
  B) 上月在巴西召开的会议上自由贸易的支持者辩称,这些经济政策会使所有国家都受益。
  C) 自由贸易的支持者在上个月召开巴西会议时提出了这些经济政策,以帮助所有国家。
  D) 在上个月的会议上,政策是否有利于所有国家展开了辩论。
  64. The game industry as part of the entertainment business is catching up fast with movies and TVs in terms of both popularity and yearly profit.
  A) 游戏业和娱乐业一样,在受欢迎的程度和年收入方面都飞快超越了电影和电视。
  B) 游戏产业作为娱乐业的组成部分,它在普及性和年收人方面都在很快超越电影和电视。
  C) 作为娱乐业的组成部分,游戏产业在其普及性和年利润方面都迅速逼近电影和电视。
  D) 游戏业和部分娱乐业如同电影和电视一样,越来越受欢迎,收入逐年迅速提高。
  65. On July 10th the company will hold its first worldwide video conference(电视会议) .All twenty facilities will be linked by a satellite broadcasting system SO that employees can see and speak with each other. Mr. Black will begin the conference by telling US about our goals for the next five years. Next,each manager will speak about current challenges. devoted to questions from all locations. If it proves successful,we hope to schedule worldwide video conferences annually.
  答案:61-64:DCBC       65:7月10日公司将举行电视会议。20个设施将通过一个卫星传播系统连接起来,这样员工就可以观看并且相互讨论。会议一开始,布莱克先生将会对公司接下来5年内的目标进行说明。接着,每一位经理会对目前面临的挑战进行阐述。最后一小时将会留给各个分支机构进行提问。如果这次成功的话,我们希望以后每年都会举行世界范围内的电视会议。
 2010年12月英语三级(A)考试模拟题(语法词汇篇1)

Directions: There are 10 incomplete statements here. You are required to complete each statement by choosing the appropriate answer from the 4 choices marked A), B), C) and D). You should mark the corresponding letter on the Answer Sheet with a single line through the center.
  16. The question is ________ me and I have no idea of it.
  A) beyond 
  B ) over 
  C) beside 
  D) above
  17. The output of our company this month is _______ that of last month.
  A) twice as much as 
  B) twice as much
  C) twice many as 
  D) twice as many as
  18. Hardly had I got home ________ the telephone rang.
  A) then 
  B) when 
  C) as 
  D) than
  19. By the time you get back, I ________ all the work.
  A) would finish 
  B) will have finished
  C) has finished 
  D) had finished
  20. It’ s required that the students ________ the term paper tomorrow.
  A) finished 
  B) finish 
  C) will finish 
  D) may finish
  21._______ the final examination is over, we can go outside for a picnic.
  A) Even though 
  B) Now that 
  C) For 
  D) With
  22. Tom was the only one of the students who ________ named Outstanding Student.
  A) is 
  B) are 
  C) was 
  D) were
  23. There is something wrong with my car. It will take me one hour to have it ______.
  A) fix 
  B) fixing 
  C) to fix 
  D) fixed
  24. Seldom ________ his wife punishes her children for speaking out their own ideas freely.
  A) I saw 
  B) I have seen
  C) have I seen 
  D) do I see
  25. It’ s high time the government _________ some measures to raise people’ s awareness of environmental protection.
  A) will take 
  B) take 
  C) took 
  D) takes

16.【答案】A【译文】我不懂这个问题,一点儿也不知道。 【考点】固定搭配 【解析】“beyond sb.”意为“为某人所不能理解,超出某人的能力范围”,其他词均无此搭配,故选A。
17.【答案】A【译文】我们工厂这月的产量是上月的两倍。  【考点】倍数的表达法
【解析】倍数+as much/many as,用much还是many由主语的形式决定。

18.【答案】B【译文】我一到家电话铃就响了。  【考点】固定搭配
【解析】hardly…when是固定关联词组,意为“一…就…”,类似的词组还有“scarcely…when…”,“no sooner…than…”等。
19.【答案】B【译文】到你回来的时候,我将已完成所有的工作。 【考点】动词的时态
【解析】by引导的时间短语通常和完成时态连用。“by the time you get back”表示将来的时间,因此主句应用将来完成时态,故选B。
20,【答案】B【译文】要求学生们明天完成学期论文。 【考点】虚拟语气
【解析】本题考查的是虚拟语气在主语从句中的用法。其结构为it+形容词或过去分词+主语从句,用虚拟语气,即“should+动词原形”(should可以省略)。类似的词有“essential,urgent,natural,important,desirable,necessary,ordered”等。
21.【答案】B【译文】既然期末考试已经结束,那么我们可以外出野餐了。【考点】状语从句【解析】even though引导让步状语从句,表“似乎,好像”;now that引导原因状语从句,表“既然”。
22.【答案】C【译文】汤姆是这些学生中唯一一名被授予优秀学生称号的人。 【考点】主谓一致 【解析】the only one of+名词,后面的定语从句中的谓语动词用单数形式,但one of+名词后面的定语从句中的谓语动词用复数,又因题中主句时态为一般过去时,故此题选C。 
  23.【答集】D
  【译文】我的车出问题了,得花费我一小时时间找人修理。
  【考点】非谓语动词
  【解析】本题考查的是使役动词have后接过去分词作宾语补足语表被动的用法。此类动词还有:keep.find,leave,make,get等。
  24.【答案】C
  【译文】我很少看到他妻子因为孩子自由地说出自己的想法而惩罚孩子。
  【考点】倒装结构及时态
  【解析】具有否定意义或否定形式的词或词组位于句首时,谓语用部分倒装结构。其他具有否定意义的词还有:never,little,scarcely,hardly,not only等。另外,根据句意应用现在完成时态。
  25.【答案】C
  【译文】现在是时候政府采取措施提高人们的环保意识了。
  【考点】虚拟语气
  【解析】当It’s(high/about)time…后接从句时,从句谓语动词用过去时,意为“早该干某事而现在做已经有些晚了”。

2010年12月英语三级(A)考试模拟题(语法词汇篇2)

Directions: There are 10 incomplete statements here. You are required to complete each statement by choosing the appropriate answer from the 4 choices marked A), B), C) and D). You should mark the corresponding letter on the Answer Sheet with a single line through the center.
  16. All students are eager to know ________they wilt pass the exam or not.
  A) if B) that C) whether D) why
  17. When the couple _______ here, they ________ to the park.
  A) will come;will go B) come;go
  C) will come;go D) come;will go
  18. —I cannot find Tom anywhere.
  —He _________ have been off long. I heard him make a call just now.
  A) shouldn’t B) can’t C) mustn’t D) needn’t
  19. No matter who he is, young or old, people’s state of mind tends to keep _______ with the rapid change of society.
  A) contact B) progress C) touch D) pace
  20. To tell you the truth, it’s very hard for us to help them get rid of Internet addiction. But we _________ on this problem trying to improve the situation.
  A) worked B) had worked C) are working D) had been working
  21. There was an earthquake happened, _______ 100 people and with more than 300
  A) killed; injured B) killing; injured
  C) killed; injuring D) killing; injuring
  22. 50miles southwest of the country ________ the famous beautiful mountain.
  A) laying B) lie C) lies D)lays
  23. Everyone has periods in their lives ________ everything seems so hard.
  A) when B) where C) which D) that
  24. Miss Liu doesn’t mind _______ your homework as long as it is done by yourself.
  A) you to delay handing in B) your delaying handing in
  C) your delaying to hand in D) you delay to hand in
  25. By no means _______ ready to quit her study.
  A) will she B) she will C) she is D) is she

16.【答案】C
  【译文】所有学生都十分渴望知道他们能否通过这次考试。
  【考点】连词辨析用法
  【解析】根据句意可以排除选项B和D。虽然连词if和whether都有“是否”的含义,但只有whether能与or not搭配。
  17.【答案】D
  【译文】当这对夫妻来到这儿时,他们会去公园。
  【考点】动词时态
  【解析】在这句中they _______to the park.是主句,When the couple _______ here是从句。当主句是将来时态或表示将来意义时,从句用一般现在时代替将来时。
  18.【答案】B
  【译文】“我在哪儿都找不到汤姆。”“他一定没走多远,我刚才还听见他打电话了呢”。
  【考点】考查情态动词与虚拟语气用法
  【解析】语境表示语气很肯定的否定判断推测:我刚才还听见他打电话了,因此他一定没走多远。用can’t have done。shouldn’t have done表示本不应该做某事;mustn’t have done表示禁止做某事;而needn’t have done指的是本来没必要做某事。
  19.【答案】D
  【译文】无论老少,人们的想法都能与时俱进。
  【考点】动词短语
  【解析】选项A与keep…with搭配时应为keep in contact with。选项B无法与keep…with搭配。选项C,keep touch with意为与…保持联系。只有keep pace with符合题意,意为与…并驾齐驱。
  20.【答案】C
  【译文】说实话,帮助他们戒除网瘾很困难,但是我们正在努力解决这个问题。
  【考点】动词时态
  【解析】根据语境意义,我们可知这个动作正在进行中,因此用现在进行时。
  21.【答案】B
  【译文】最近发生了一次地震,造成死亡l00人,300多人受伤。
  【考点】非谓语动词
  【解析】第一空用kill的现在分词作状语,表示主动含义和补充说明;第二空用过去分词后置“300(people)”,意为“受伤的”。
  22.【答案】C
  【译文】这个国家西南部50英里的地方坐落着一座美丽的高山。
  【考点】倒装结构及主谓一致
  【解析】首先要知道此句为倒装结构,另外根据句意可知,主语为“the famous beautiful mountain”,指的是一座漂亮的山,所以谓语也应该用单数。
  23.【答案】A
  【译文】每个人的生命里都有一段什么事情都看似十分艰难的时期。
  【考点】定语从句
  【解析】本题的关系词引导定语从句修饰periods,指的是一段时期,并在从句中作时间状语,因此选A。
  24.【答案】B 
  【译文】只要你的作业是自己做的,刘老师并不介意你晚些再交上来。
  【考点】动名词用法
  【解析】mind和delay后都要接动名词形式。因此选B。
  25.【答案】D
  【译文】她从来也没准备要辍学。
  【考点】倒装结构
  【解析】当否定词或含有否定意义的词、短语放在句首时,句子一般要部分倒装,因此只能选择B或D。但因为be ready to do为固定搭配,意为“准备做某事”,故选D;B项缺少谓语。

2010年12月英语三级(A)考试模拟题(语法词汇篇3)

Directions: In this section, there are 10 incomplete sentences. You are required to complete each one by deciding on the most appropriate word or words from the 4 choices marked A), B), C) and D).Then you should mark the corresponding letter on the Answer Sheet with a single line through the center. 
  16. We’11 never forget ______ in New York last year.
  A) us to meet a few friends   B)our meeting a few friends
  C) a few friends to meet   D) a few friends meeting
  17. Is there any hope of ______ the gold medal?
  A) John to win   B) John win
  C) winning John   D) John’ s winning
  18. There is a man at the reception desk who seems very angry and I think he means _______ trouble.
  A) making   B) to make
  C) to have made   D) having made
  19._______ her surprise, Miss Wang found nobody in the classroom.
  A) At   B) To
  C) For   D) With
  20. Anderson is one of those _______ appear friendly but in fact are hard to deal with.
  A) who   B) they
  C) that   D) which
  21. I wish they _______ in their ball game.
  A) will be succeeded   B) succeeded
  C) would be successful   D) were success
  22. It is difficult for her to make the final decision now, _______ she is in a delicate situation.
  A) if   B) unless   C) until   D) since
  23. In old China women used _______.
  A) to look down   B) to look down upon
  C) to be looked down   D) to be looked down upon
  24. I suppose you know everything about that event, ________ ?
  A) don’ t you   B) do I
  C)do you   D) don’ t I
  25. Hellen is fond of light music, _______ her brother is keen on rock and roll.
  A) when   B) and
  C) while   D) for

16.【答案】B 
  【译文】我们永远不会忘记去年那次在纽约与几个朋友的会面。 【考点】固定用法
  【解析】forget后面既可以接不定式,即forget to do sth,意为“忘记要做某事”;又可以接动名词,即forget doing sth,意为“忘记做过某事”。根据句意,故选B。
  17.【答案】D
  【译文】约翰有希望赢得金牌吗?【考点】非谓语动词 【解析】如果动名词动作的发出者不是句子的主语时,则需要有自己的逻辑主语,其形式为:物主代词+动名词;名词或人称代词宾格+动名词;名词’s+动名词。根据题意,故选D。
  18.【答案】B
  【译文】接待处有个男人看起来非常生气,我想他要找麻烦。 【考点】固定用法
 【解析】mean to do意为“打算做某事”;mean doing意为“意味着…”。根据句意,故选B。
  19.【答案】B
  【译文】令王老师惊讶的是,她发现教室里一个人也没有。 【考点】固定搭配
  【解析】to one’s surprise为固定搭配,意为“令某人惊讶的是”。故选B。
  20.【答案】A
  【译文】安得森是外表和蔼而实际上却很难相处的人中的一个。 【考点】定语从句
  【解新】这里需要一个表示人的关系代词,四个词中,只有who最合适。故选A。
  21.【答案】B
  【译文】我希望他们在球赛中取得成功。 【考点】虚拟语气 【解析】wish后面的从句要用虚拟语气,这里是现在时,所以从句要用动词的过去式,故选B。
  22.【答案】D
  【译文】因为她处境微妙,所以现在很难作最后的决定。 【考点】状语从句
  【解析】四个词中,只有since可以引导原因状语从句,表示“由于,因为”的意思,故选D。
  23.【答案】D
  【译文】在旧中国妇女常常被人瞧不起。
  【考点】被动语态
  【解析】used to do sth表示“过去常常做某事”,而根据句意可知,women与谓语动词之间是被动关系,look down upon表示“瞧不起”,这里应该用它的被动语态,故选D。
  24.【答案】A
  【译文】我想你了解事件的一切,是吗?
  【考点】反意疑问句
  【解析】当主句谓语是think,believe,expect,suppose,imagine等引导的宾语从句时,疑问部分应与宾语从 句相对应构成反意疑问句。所以应忽略主句I suppose,根据从句来选择。从句是肯定句,主语是you,故选A。
  25.【答案】C
  【译文】海伦喜欢轻音乐,而她弟弟却热衷于摇滚乐。
  【考点】连词的用法
  【解析】四个可供选择的连词中,只有while可表示对比之意,有“而…”的意思。故选C。

2010年12月英语三级(A)考试模拟题(语法词汇篇4)

Directions: In this section, there are 10 incomplete sentences. You are required to complete each one by deciding on the most appropriate word or words from the 4 choices marked A), B), C) and D).Then you should mark the corresponding letter on the Answer Sheet with a single line through the center.
  16._________,Gloria started looking for work.
  A) All the money being spent  B) All the money having been spent
  C) All the money spend  D) Spending all the money
  17. It will take you one hour and fifty minutes _________ the top of Mount Tai.
  A) reaching  B) to reach
  C) being reached  D) having been reached
  18._________ the news, Ida changed her plans.
  A) In hearing  B) To hear
  C) Of hearing   D) On hearing
  19. Tony is talking about the friends and the food _________ interest him.
  A) which  B) who
  C) that  D)as
  20. I didn’t understand at first why he couldn’t pass the test, no matter _________ hard he had tried.
  A) when  B) where
  C) what  D) how
  21. In the ________ half of the 17th century those cities became even more prosperous.
  A) latter  B) latest
  C) later  D) late
  22. ________ coming here, I haven’t seen a single person.
  A) Since  B) From
  C) By  D) At
  23. Between 1975 and 1996, the number of overseas visitors expanded _________ 27% on average each year.
  A) in  B) by  C) for  D) to
  24. So little ________ about chemistry that the lecture was completely beyond me.
  A) did I know  B) I know
  C) do I know  D) I knew
  25. Had I known the result, I _________ you to tell me.
  A) would not ask  B) would not have asked
  C) had not asked  D) have not asked

 16.【答案】B
  【译文】钱全部花光之后,格洛丽娅开始找工作。
  【考点】独立主格结构
  【解析】这是非谓语动词中分词的独立主格结构。分词动作发生在主句谓语动词之前,因此要用完成时,故选B。
  17.【答案】B
  【译文】你得花1小时50分钟才能爬到泰山顶上。
  【考点】非谓语动词
  【解析】不定式(短语)或不定式复合结构for sb.to do做主语时,往往将其移至谓语动词之后,将it置于句首作形式主语。take sb.+时间(金钱、精力)+to do表示“花费…时间(金钱、精力)做某事”,故选B。
  18.【答案】D
  【译文】一听到那消息,艾达便改变了计划。
  【考点】非谓语动词
  【解析】“on(upon)doing”表示“一…就…”。根据题意,故选D。
  19.【答案】C
  【译文】托尼正在谈论他感兴趣的朋友和食物。
  【考点】定语从句
  【解析】当定语从句的先行词中既有表示人的名词又有表示物的名词时,只能用that来引导,故选C。
  20.【答案】D
  【译文】起初我不明白为什么无论他怎么努力都考不及格。
  【考点】状语从句
  【解析】how修饰副词hard,no matter how引导让步状语从句,故选D。
  21.【答案】A
  【译文】在l7世纪的后半叶,那些城市变得更加繁华。
  【考点】形容词的用法
  【解析】latter意为“后半的”,如:the latter half of the month意为“后半月”,故选A。
  22.【答案】A
  【译文】从我来到这里开始就没有见过一个人。
  【考点】连词的用法
  【解析】因为“I haven’t seen a single person”是现在完成时,所以需要和表示时间段的连词短语连用。而coming here是一个时间点,四个词中,只有since适合,故选A。
  23.【答案】B
  【译文】在1975到1996年间,海外游客的数量以平均每年27%的速度增长。
  【考点】介词的用法
  【解析】expand by…%表示“以某种百分比增长”,故选B。
  24.【答案】A
  【译文】我对化学了解甚少,所以这个演讲我完全听不懂。
  【考点】倒装句
  【解析】little相当于一个否定词,与副词so放在句首时,句子需要部分倒装,另外助动词do的时态要与that从句的时态保持一致,故选A。
  25.【答案】B
  【译文】要是我早知道这个结果,我就不会要你告诉我了。
  【考点】虚拟语气
  【解析】这是虚拟语气在条件状语从句中的应用。从句是省略if的条件状语从句,用过去完成时“had known”表示对过去的假设,主句要选择“would have done”结构,故选B。

2010年12月英语三级(A)考试模拟题(语法词汇篇5)

Directions: In this section, there are 10 incomplete sentences. You are required to complete each one by deciding on the most appropriate word or words from the 4 choices marked A), B), C) and D).Then you should mark the corresponding letter on the Answer Sheet with a single line through the center.
  16. She moved away from her parents, and missed them __________ enjoy the exciting life in New York.
  A) too much to 
  B) enough to
  C) very much to 
  D) so much to
  17. __________, the negotiation went further on.
  A) With the first point agreeing on 
  B) With the first point agreed on
  C) With the first point to agree on 
  D) With agreeing on the first point
  18. I regret __________ the novel to Jim because he made it very dirty.
  A) to lend 
  B) having lent
  C) to be lending 
  D) lent
  19. The work they are doing is __________ great value.
  A) in 
  B) of 
  C) by 
  D) from
  20. The doctor insisted that Paul’s mother __________ examined thoroughly.
  A) would be 
  B) must be 
  C) be 
  D) should have been
  21. Kevin is the only student in the class __________ failed in the exam.
  A) who 
  B) that 
  C) which 
  D) whom
  22. The projects __________ by the end of 2002 will expand the city’s telephone network to cover 1,000,000 users.
  A) being accomplished 
  B) to be accomplished
  C) accomplished 
  D) having been accomplished
  23. There is no doubt __________Herbert is the most industrious student in our class.
  A) whether 
  B) what 
  C) that 
  D) if
  24. Up went the prices and _________
  A) the living standard came down 
  B) down did the living standard come
  C) came down the living standard 
  D) down came the living standard
  25. The ratio of the work done by the machine _________ the work done on it is called the efficiency of the machine.
  A) against 
  B) with 
  C) to 
  D) for

16.【答案】A
  【译文】她离开父母只身搬到纽约,然而却由于太想念他们而无法享受NJL的精彩生活。
  【考点】固定搭配
  【解析】“too…to”为固定搭配,表示否定的含义,意为“太…以至于不…”;“enough…to”结构则表示肯定含义,意为“足够能…”。注意enough用于名词之前,而用于形容词或副词之后,故选A。
  17.【答案】B
  【译文】大家就第一条款达成一致意见后,谈判继续进行了。
  【考点】独立主格
  【解析】分词的独立主格结构的构成还包括:with/without+名词/代词+分词/形容词/副词/介词短语。本题中the first point与agree on是被动关系,所以要用过去分词,故选B。
  18.【答案】B 
  【译文】我后悔把这本小说借给了吉姆,因为他把书弄脏了。
  【考点】固定搭配
  【解析】regret后面既可以接不定式,又可以跟动名词,前者指“对尚未做,或正在做的事情的遗憾”,后者指“对过去做过的事情的遗憾”。根据句意,故选B。
  19.【答案】B
  【译文】他们正从事的工作意义重大。
  【考点】介词的用法
  【解析】be of+抽象名词相当于该抽象名词的形容词,故选B。
  20.【答案】C
  【译文】医生坚持要保罗的母亲做全面检查。
  【考点】虚拟语气
  【解析】insist表示坚持观点时,不用虚拟语气,如:He insisted that he had done nothing wrong.他坚持认为他没有做错。但是表示坚持要求某种做法时,则用虚拟语气,故选C。
  21.【答案】B
  【译文】凯文是班上这次考试惟一没及格的学生。
  【考点】定语从句
  【解析】当定语从句的先行词前有the only,me very,Ⅱle last,the same等表示特定概念的修饰词时,定语从句的引导词只能用that,故选B。
  22.【答案】B
  【译文】于2002年底完成的这项工程将会使城市的电话网络覆盖到1,000,000名用户。
  【考点】时态和语态
  【解析】此句谓语动词为将来时,根据题意,所填词需作project的定语,所以我们选不定式作其定语表示将来;而accomplish与project之间又有动宾关系,故选择不定式的被动语态,故选B。
  23.【答案】C
  【译文】赫伯特是我们班最勤奋的学生,这一点是不用怀疑的。
  【考点】同位语从句
  【解析】名词doubt用在肯定句中,其后的同位语从句用whether引导,若用在否定句中则由that引导,故选C。
  24.【答案】D
  【译文】随着物价的上涨,生活水平在下降。
  【考点】倒装结构
  【解析】此句将副词up和down置于句首,句子应全部倒装,故选D。
  25.【答案】C
  【译文】机器所做的工作与在机器上所用的功之比,就称为机器的效率。
  【考点】固定搭配
  【解析】题中ratio与介词to是固定搭配,the ratio of A to B的意思为“A与B之比(率)”,故选C

2010年12月英语三级(A)考试模拟题(语法词汇篇6)

 Directions: In this section, there are 10 incomplete sentences. You are required to complete each one by deciding on the most appropriate word or words from the 4 choices marked A), B), C) and D) .Then you should mark the corresponding letter on the Answer Sheet with a single line through the center.
  16. Is there any hope of ________ the final exam?
  A) John to pass 
  B) John pass
  C) passing John
  D) John’s passing
  17. Lawrence will do anything for Lily except _________ her money.
  A) lending 
  B) lend 
  C) borrowing 
  D) borrow
  18. We found it impossible ________ all the questions within the time given.
  A) solve 
  B) being solving 
  C) to have solved 
  D) to solve
  19. We don’t allow _________ in the meeting room.
  A) to smoke 
  B) smoked 
  C) smoking 
  D) to smoking
  20. ________ is known to the world, Mark Twain is a great American writer.
  A) As 
  B) That 
  C) Who 
  D) Which
  21. There were _________ for Saturday’s movie.
  A) not available tickets 
  B )no available tickets
  C) no tickets available 
  D) tickets no available
  22. The output of our factory is now twice ________ it was three years ago.
  A) what 
  B) that 
  C) as 
  D) which
  23. At no time and under no circumstances ________ the first to use nuclear weapons.
  A) China will be 
  B) will China be
  C) will be China 
  D) shall China be
  24. What ________ would happen if the manager knew you felt that way?
  A) will you suppose 
  B)are you supposing
  C) do you suppose 
  D) you would suppose
  25. It is high time someone ________ Nick that each member of a team has to do his share of work.
  A) could tell 
  B) tell
  C) told 
  D) would tell

16.【答案】D 【译文】约翰有希望通过期末考试吗?【考点】非谓语动词 【解析】如果动名词动作的发出者不是句子中谓语动词的主语时,则需要有自己的逻辑主语,其形式为:物主代词+动名词:名词或人称代词宾格+动名词;名词’s+动名词,故选D。
  17.【答案】B【译文】除了借钱,劳伦斯会为莉莉做任何事。 【考点】固定用法 【解析】不定式(短语)作介词except的宾语,如果主句中有do,did或does,except后就接不带to的不定式,反之则接带to的不定式,故选B。

18.【答案】D【译文】我们发现,要想在规定的时间内答完所有的题目是不可能的。【译文】非谓语动词 【解析】不定式(短语)做宾语时,如果其后跟补足语,通常用it作形式宾语来替代不定式这个真正宾语,故选D。
  19.【答案】C【译文】会议室内不允许吸烟。  【考点】固定搭配 【解析】allow+动名词或者allow+sb.+to do sth.为固定搭配,都表示“允许做某事”,故选C。
  20.【答案】A【译文】众所周知,马克·吐温是一位伟大的美国作家。  【考点】定语从句 【解析】as作关系代词置于句首引导非限定性定语从句,修饰后面的主句,表示“(正)如…,像…”之意,故选A。
  21.【答案】C【译文】星期六的电影票没有了。 【考点】后置定语 【解析】形容词available作定语修饰名词时通常放在名词之后作后置定语,故选C。
  22.【答案】A【译文】我们工厂现在的产量是它三年前的两倍。  【考点】比较倍数的表达 【解析】倍数的表达方式可以是倍数+名词,此句中,名词是一个由what引导的名词从句,what是从句的表语,故选A。
  23.【答案】B【译文】在任何时候和任何情况下,中国都决不会第一个使用核武器。 【考点】倒装句 【解析】由具有否定意义的副词或短语置于句首的句子要部分倒装,即将助动词或情态动词等提到主语前,故选B。
  24.【答案】C 【译文】你猜想如果经理知道你是那样想会怎么样?【考点】插入语  【解析】do you suppose放在此句中作为插入语,故选C。
  25.【答案】C【译文】是该告诉尼克队里的每一个成员都必须要分担工作的时候了。 【考点】虚拟语气 【解析】在句型It is high time+定语从句中,要用虚拟语气,所以从句用过去时,故选C。

2010年12月英语三级(A)考试模拟题(语法词汇篇7)

Directions: In this section, there are 10 incomplete sentences. You are required to complete each one by deciding on the most appropriate word or words from the 4 choices marked A), B), C) and D). Then you should mark the corresponding letter on the Answer Sheet with a single line through the center.
  16. Laptops are now so expensive ________ beyond the reach of almost everyone.
  A) that be 
  B) as to be 
  C) as be 
  D) as being
  17. You might as well ________ your money as spend it in gambling.
  A) to throw 
  B) throwing 
  C) throw 
  D) to be thrown
  18. ________ his appearance, he comes from a well-off family.
  A) Judged from 
  B) To judge from
  C) Judging from 
  D) Having been judging from
  19. He must have heard of the bad news, ________ can be perceived in his expression.
  A) as 
  B) what 
  C) while 
  D) that
  20. At the beginning he refused to take any responsibility but he had to end up by ________.
  A) to apologize 
  B) apologizing
  C) apologized 
  D) apologize
  21. With the development of artificial intelligence, robots will do some mental work in office ________ some manual work on the production line.
  A) in addition 
  B) except
  C)as well as 
  D)as well
  22. My plane ________ at 9 o’ clock tomorrow morning. Will you come to see me off?
  A) will be leave 
  B) leaves
  C) will have left 
  D) is left
  23. To know what is good and ________ are two different things.
  A) doing what is right 
  B)to do what is right
  C) did what was right 
  D) to do what was right
  24. They have to adjust in ________ way that they don’ t make a loss.
  A) a such 
  B ) such 
  C) such a 
  D) so
  25. We are all for your suggestion that the trip ________
  A) is to put off 
  B )was put off
  C) should put off 
  D) be put off

16.【答案】B
  【译文】现在笔记本电脑还很贵,不是人人都买得起。
  【考点】固定搭配
  【解析】“so+形容词/副词+as+不定式”属于固定搭配,表示“如此…以至于”,故选B。
  17.【答案】C 
  【译文】你与其把钱花在赌博上,不如把它丢掉的好。
  【考点】固定搭配
  【解析】英语中有些固定的搭配如:had better,would rather...than,would…rather than,cannot but,cannot help but,may as well等后要接不带to的不定式,故选C。
  18.【答案】C
  【译文】从他的外表来判断,他来自富有的家庭。
  【考点】非谓语动词
  【解析】题目中分词的逻辑主语与句子的主语虽然不一致,但是也不能用过去分词,本题属于某些固定结构judging from/by…,表示“从…来判断”;generally/frankly speaking…,表示“一般/坦白地说…”,故选C。
  19.【答案】A
  【译文】从他的表情可以看出来,他一定是听说了这个消息。
  【考点】定语从句
  【解析】as在这里作关系代词,引导非限定性定语从句,说明前面整句话,符合题意,故选A。
  20.【答案】B
  【译文】起初他拒绝承担任何责任,但最终只好道歉。
  【考点】介词的用法
  【解析】apologizin9动名词作介词by的宾语,故选B。
  21.【答案】C
  【译文】随着人工智能的发展,机器人除了在生产线上做一些手工劳动外,还将在办公室做一些脑力劳动。
  【考点】连词的用法
  【解析】根据题意,需要一个并列连词将manual work与mental work连接起来,共同作do的宾语,as well as此处为“除…之外;同;和”之意,符合题意。其他选项中,as well意为“也”;except意为“除了…(不包括在内)”;in addition意为“此外”,均不合题意。故选C。
  22.【答案】B
  【译文】我的飞机明天9点起飞,你来送我吗?
  【考点】时态
  【解析】一般现在时可与表示将来时间的状语连用,表示按计划、安排会发生的情况,此句用leave的一般现在时就是表示此意的用法。一般这种用法只限于某些表示移动的动词,如:go,come,leave,start,arrive,depart,stop,begin,close等,故选B。
  23.【答案】B
  【译文】知道什么是应该做的和去做应该做的事是两件完全不同的事情。
  【考点】主语从句
  【解析】to do what is right与句前的to know what is good为对称平衡结构,共同作并列主语,故选B。
  24.【答案】C
  【译文】他们不得不调整方法以便不会亏损。
  【考点】状语从句连词 
  【解析】such修饰名词时,不定冠词应放在名词之前,不用a such…或such...,so在这里不妥,故选C。
  25.【答案】D
  【译文】我们都赞成你的建议,将旅行延期。
  【考点】同位语从句和虚拟语气
  【解析】句中suggestion要求其后的同位语从句的谓语动词用“(should)+动词原形”的结构,故选D。

2010年12月英语三级(A)考试模拟题(语法词汇篇8)

Directions: In this section, there are 10 incomplete sentences. You are required to complete each one by deciding on the most appropriate word or words from the 4 choices marked A), B), C) and D). Then you should mark the corresponding letter on the Answer Sheet with a single line through the center.
  16. In spite of what we said, he refused to ________ to the police station.
  A) give away 
  B) give off
  C) give himself away 
  D) give himself up
  17. It is imperative that the students ________ writing their papers by the end of the month.
  A) finish 
  B) must finish 
  C) will finish 
  D) are to finish
  18. I invited Tom and Ann to dinner, but ________ of them came.
  A) both 
  B) none 
  C) either 
  D) neither
  19. The heart is an important organ of circulation ________ function is to pump blood to all parts of the body.
  A) which 
  B) whose 
  C) of it 
  D) its
  20. _________ with the size of the whole earth, the highest mountain does not seem high at all.
  A) When compared 
  B) Compare
  C) While comparing 
  D) Comparing
  21. Training astronauts _________ not an easy thing.
  A) are 
  B) is 
  C) were 
  D) be
  22. Would you be _________ by the idea of going to Paris?
  A) exciting 
  B) excitably 
  C) excited 
  D) excitedly
  23. There is no ________ that women are playing an important role in the world today.
  A) of deny 
  B) to denying 
  C) denying 
  D) of denying
  24. I am rather concerned ________ her, for I haven’t heard from her since last winter.
  A) of 
  B) at 
  C) with 
  D) about
  25. Look at the terrible situation I am in! If only I ________ your advice.
  A) had followed 
  B) would follow
  C) follow 
  D) have followed

 16.【答案】D 【译文】不管我们怎么说,他都拒绝去警局自首。 【考点】动词短语的词义辨析  【解析】give sb.up意为“自首,表示对…一没有希望”。例:It was so late that we had given him up.已经很晚了,我们认为他不会来了。give away意为“送掉,分发,放弃,泄露,出卖,让步,陷下”。例:He was invited to attend the closing ceremony of the tournament and to give away the prizes.他应邀出席锦标赛的闭幕式并颁发奖品。He gave away the secret without meaning to.他无意中把秘密泄露了。give off表示“发出,放出(蒸汽、气味等)”。例:This kind of coal gives off dense smoke.这种煤烧起来直冒浓烟。根据题意,故选D。
  17.【答案】A【译文】学生必须在月底以前做完论文。 【考点】虚拟语气 【解析】imperative的意思是“必要的,强制的”。当表示建议、指令、愿望、个人意见或判断、要求时,构成It is...that...结构,主语从句要用虚拟语气,动词形式为“should+动词原形”或只用动词原形,故选A。
  18.【答案】D【译文】我邀请了Tom和Ann共进晚餐,但是他们一个都没来。 【考点】代词的用法 【解析】both指两个都,none指没有一个(通常是指三个或三个以上的范围中),either指两个中的任意一个,neither指两个都不。根据本题的意思,故选D。
  19.【答案】B【译文】心脏是血液循环的器官,它的功能是把血液输送到身体的各个部位。 【考点】定语从句 【解析】选项A的which应该在定语从句中担任一定的成分,但空格之后的句子是主系表结构,主谓成分已经很齐全,所以不适合;选项C不能使原句的结构成立;而假如把its放在空格处,在其前应该加句号将两句分开,故D也不合适;选项B的whose作关系代词时,既可指人,也可指物,符合题意,故选B。
  20.【答案】A【译文】和整个地球的大小相比,地球上最高的山看来一点儿也不高。 【考点】分词作状语 【解析】句中compare是及物动词,其动作的对象是句子的主语the highest mountain,为此须用表示被动意思的过去分词compared,而不能用现在分词comparing,故选A。  21.【答案】B【译文】训练宇航员可不是件容易的事。 【考点】主谓一致 【解析】当分词短语作主语时,谓语动词用单数,故选B。
22.【答桑】C【译文】去巴黎的想法会不会让你感到很兴奋?【考点】形容词辨析 【解析】动词变成形容词有两种形式:主动和被动。主动形式一般用来表达物的性质;被动形式一般用来表达人的态度、情绪等特征。例:The book is interesting./He is interested in politics.根据题意,故选C。
23.【答案】C【译文】女性在当今社会中发挥着相当重要的作用,这已是不争的事实。 【考点】形容词辨析 【解析】作限定词时后面接名词,相当于not a(n)或not any。denying为动名词,具有名词的性质,故选C。
24.【答案】D【译文】我十分担心她,因为自去年冬天到现在我都没有收到她的信。 【考点】固定短语 【解析】be concerned about是固定搭配,意为“担心,挂念”,故选D。

25.【答案】A【译文】瞧,我的处境多糟糕,要是听了你的主意该多好啊!【考点】虚拟语气 【解析】if only引导的句子,表示说话人的一种愿望,希望过去发生的事能与事实相反。由此可见,在本题中,事实上说话人并不曾听劝告,用过去完成时表达对过去的一种愿望,故选A。

731 Views
分享你的喜爱
linwute
linwute

我要像梦一样自由,像大地一样宽容;
在艰辛放逐的路上,点亮生命的光芒;
我要像梦一样自由,像天空一样坚强;
在曲折蜿蜒的路上,体验生命的意义;

留下评论

您的电子邮箱地址不会被公开。 必填项已用 * 标注